SlideShare a Scribd company logo
1 of 112
Download to read offline
Федеральное агентство по образованию
ГОУ ВПО Тульский государственный педагогический университет
им. Л. Н. Толстого
Р. Р. Яфаева, Ю. И. Богатырева
МАТЕМАТИКА
И ИНФОРМАТИКА
Учебно-методическое пособие
Допущено Учебно-методическим объединением
по направлениям педагогического образования
Министерства образования и науки РФ в качестве
учебного пособия для студентов высших учебных заведений,
обучающихся по направлению 050700 «Педагогика»
В 2 частях
Часть 2: ПРАКТИКУМ
Тула
Издательство ТГПУ им. Л.Н. Толстого
2010
Copyright ОАО «ЦКБ «БИБКОМ» & ООО «Aгентство Kнига-Cервис»
2
ББК 22.1я73+32.81я73
Я89
Рецензенты:
доктор физико-математических наук, профессор В. И. Желтков
(Тульский государственный университет);
кандидат педагогических наук, доцент О. В. Чукаев
(Тульский государственный педагогический
университет им. Л. Н. Толстого)
Яфаева, Р. Р.
Я89 Математика и информатика: Учеб.-метод. пособие: В 2 ч.
Ч. 2: Практикум / Р. Р. Яфаева, Ю. И. Богатырева.– Тула: Изд-во
Тул. гос. пед. ун-та им. Л. Н. Толстого, 2010.– 111 с.
В пособии представлены основные положения дисциплины «Математика и
информатика», адаптированные для студентов направления подготовки «Педа-
гогика». Математика представлена следующими разделами: аксиоматический
метод построения математических теорий, комбинаторика, теория множеств,
понятия и свойства вероятностей, элементы математической статистики.
Информатика представлена разделами: понятие, свойства и измерение инфор-
мации; алгоритмы и языки программирования; понятие и компоненты про-
граммного и аппаратного обеспечения современной компьютерной техники.
Практические задания направлены на формирование умений использовать
современные информационные технологии и стандартное программное обеспе-
чение в профессиональной деятельности педагога. Представленные примеры
решения задач позволяют использовать пособие для организации самостоятель-
ной работы студентов.
Для студентов высших учебных заведений, обучающихся по направлению
подготовки 050700 «Педагогика».
ББК 22.1я73+32.81я73
© Р. Р. Яфаева, Ю. И. Богатырева, 2010
© Издательство
ТГПУ им. Л. Н. Толстого, 2010
Copyright ОАО «ЦКБ «БИБКОМ» & ООО «Aгентство Kнига-Cервис»
3
II. Практикум
Системы счисления
Основные умения: осуществлять действия с числами в
различных системах счисления.
Рекомендуемая литература: [13, 15, 22].
Системы счисления (нумерация) – совокупность способов
обозначения натуральных чисел.
На ранних ступенях развития общества люди почти не умели
считать. Они различали совокупности двух и трех предметов; всякая
совокупность, содержавшая бóльшее число предметов, объединялась в
понятии «много». Первоначально натуральные числа изображались с
помощью некоторого количества черточек или палочек, затем для их
изображения стали использовать буквы или специальные знаки. В
древнем Новгороде использовалась славянская система, где применялись
буквы славянского алфавита; при изображении чисел над ними ставился
знак ~ (титло).
Древние римляне пользовались нумерацией, сохраняющейся до
настоящего времени под именем «римской нумерации», в которой числа
изображаются буквами латинского алфавита. Сейчас ею пользуются для
обозначения юбилейных дат, нумерации некоторых страниц книги
(например, страниц предисловия), глав в книгах, строф в стихотворениях.
Выполнение арифметических действий над многозначными числами в
этой записи очень трудно. Тем не менее, римская нумерация преобладала
в Италии до XIII в., а в других странах Западной Европы – до XVI в.
Этим системам свойственны два недостатка, которые привели к их
вытеснению другими: необходимость большого числа различных знаков,
особенно для изображения больших чисел, и, что еще важнее, неудобство
выполнения арифметических операций.
Более удобной и общепринятой и наиболее распространенной
является десятичная система счисления, которая была изобретена в
Индии, заимствована там арабами и затем через некоторое время пришла
в Европу. Существовали системы счисления и с другими основаниями.
Широкое распространение имела в древности и двенадцатеричная
система, происхождение которой, вероятно, связано, как и десятичной
системы, со счетом на пальцах: за единицу счета принимались фаланги
(отдельные суставы) четырех пальцев одной руки, которые при счете
Copyright ОАО «ЦКБ «БИБКОМ» & ООО «Aгентство Kнига-Cервис»
4
перебирались большим пальцем той же руки. Самой молодой системой
счисления по праву можно считать двоичную.
Различные группы систем счисления см. в части 1: Лекции стр. 12.
В непозиционных системах счисления от положения цифры в
записи числа не зависит величина, которую она обозначает. Примером
непозиционной системы счисления является римская система, в которой
в качестве цифр используются латинские буквы.
В позиционных системах счисления величина, обозначаемая
цифрой в записи числа, зависит от ее позиции. Количество используемых
цифр называется основанием системы счисления.
Значение числа Х, представленного в виде sp … s1s0, s-1…s-q, равно
Х=sp t р
+sp-1 t p-1
+…+ s1t + s0 + s-1t -1
+…+s-q t -q
,
где t – основание системы счисления, равное числу цифр, используемых
для записи, si – цифра.
Рассмотрим системы счисления:
1) десятичную – t=10, si∈{0,1,…,9};
2) двоичную – t=2, si∈{0,1};
3) восьмеричную – t=8, si∈{0,1,…,7};
4) шестнадцатеричную – t=16, si∈{0,1,…,9,A,B,C,D,E,F};
(416,3)8=4⋅102
+1⋅101
+6⋅100
+3⋅10-1
=400+10+6+0,3= 416,3
(10100)2 = 1⋅24
+0⋅23
+1⋅22
+0⋅21
+0⋅20
=16+4= 20
Перевод в двоичную систему счисления из 10-ичной производится
отдельно целой и дробной части.
Целая часть – последовательным делением на 2. Остатки от
деления, записанные в обратном порядке, образуют новую запись
исходного целого числа.
Пример 1: Перевести число 92 из десятичной системы в
двоичную.
92 : 2 = 46 (ост. 0)
46 : 2 = 23 (ост. 0)
23 : 2 = 11 (ост. 1)
11 : 2 = 5 (ост. 1)
5 : 2 = 2 (ост. 1)
2 : 2 = 1 (ост. 0)
9210=10111002
Дробная часть – последовательным умножением на 2. Цифры в
разряде целых образуют искомое представление исходного числа.
Copyright ОАО «ЦКБ «БИБКОМ» & ООО «Aгентство Kнига-Cервис»
5
Пример 2: Перевести число 0,648 из десятичной системы в
двоичную:
0,648 × 2 = 1,296 (1)
0,296 × 2 = 0,592 (0)
0,592 × 2 = 1,184 (1)
0,184 × 2 = 0,368 (0)
0,368 × 2 = 0,736 (0)
0,736 × 2 = 1,472 (1) и т.д.
(0,648)10=(0,101001…)2
Если основание системы счисления k можно представить в виде
k=pn
, то каждую цифру в записи числа с основанием счисления k
заменяется n цифрами системы счисления p. Рассмотрим связь между
двоичной, четырехричной, восьмеричной и шестнадцатеричной
системами счисления.
четырехричная шестнадцатеричная
Цифра двоичный код цифра двоичный код
0 00 0 0000
1 01 1 0001
2 10 2 0010
3 11 3 0011
4 0100
5 0101
восьмеричная 6 0110
0 000 7 0111
1 001 8 1000
2 010 9 1001
3 011 A 1010
4 100 B 1011
5 101 C 1100
6 110 D 1101
7 111 E 1110
F 1111
Пример 3: Представим число 92,64810 в 4-й, 8-й и 16-й системах
счисления.
В двоичной системе число 92,64810 = 1011100,1010012. Число
разбивается на группы, группы отчитываются от запятой, разделяющей
целую и дробную часть, недостающие позиции заменяются нулями. В
начале целой части или в конце дробной нули незначащие.
Copyright ОАО «ЦКБ «БИБКОМ» & ООО «Aгентство Kнига-Cервис»
6
Разобьем на пары:
1011100,1010012 = 01 01 11 00,10 10 01 = 1130,2214
Разобьем на триады:
1011100,1010012 = 001 011 100,101 001 = 134,518
Разобьем на тетроды:
1011100,1010012 = 0101 1100,1010 0100 = 5С,A416
92, 64810 = 1011100,1010012= 1130,2214= 134,518 =5С,A416
Можно заметить, что чем больше основание системы счисления,
тем меньше необходимо цифр для записи числа.
Арифметические операции
Арифметические операции во всех позиционных системах
счисления выполняются по правилам, аналогичным в десятичной
системе. Для выполнения операций необходимы следующие понятия:
A M B=C (mod D)
1) остаток от деления (D); 2) целочисленное деление (C).
Например, 8 M 5 =1 (mod 3); 10 M 3 = 3 (mod 1); 8 M 4 = 2 (mod 0).
Сложение
При сложении, числа записываются столбиком в соответствии с
разрядами. Складываются цифры. Записывается цифра, равная остатку от
деления суммы на основание системы счисления, а число, равное
результату целочисленного деления суммы на основание системы
счисления переносится в старший разряд.
При сложении чисел можно воспользоваться таблицей:
+ 0 1
0 0 1
1 1 10
Пример 4: Сложить числа 110110102 и 1010102.
Решение.
Проверка: 110110102 =218; 1010102= 42; 1000001002=260; 218+42=260
Выполнить действия: 1000112+101112=1110102; 101112 + 1012=111002;
110110102 + 1010102=1000001002; 101,012 + 11,112=10012;
4758+5148=12118; 728 + 128=1048; 16,248 + 53,458=71,718 .
1 1 1 1 1
1 1 0 1 1 0 1 02
+
1 0 1 0 1 02
1 0 0 0 0 0 1 0 02
Copyright ОАО «ЦКБ «БИБКОМ» & ООО «Aгентство Kнига-Cервис»
7
Вычитание
Для осуществления операции вычитания можно воспользоваться
таблицей сложения. Возможен заем числа равного основанию системы
счисления из старших разрядов, при этом нули, стоящие между цифрой, от
которой вычитают и цифрой, у которой «занимают» превращаются в
максимальную цифру системы счисления.
Пример 5: Вычесть из числа 11100112 число 11102.
Решение.
. 1 2
1 1 1 0 0 1 12
–
1 1 1 02
1 1 0 0 1 0 12
Проверка: 11100112= 115; 11102 = 14; 11001012=101; 115-14=101
Выполнить действия: 10101112 -11012 =10010102; 5148 – 4758=178; 728 –
168=548; 53,158 – 16,268=34,678
Умножение
При умножении числа записываются столбиком, перемножается
каждая цифра множителя на каждую цифру множимого числа, записывается
остаток от деления результата умножения на основание системы счисления,
а целая часть складывается с результатом умножения следующей цифры,
затем полученный столбец чисел складывается. Для умножения чисел можно
воспользоваться таблицей:
Двоичная система:
× 0 1
0 0 0
1 0 1
Пример 6: Найти произведение чисел 110112 и 1102.
Решение.
1 1 0 1 12
х
1 1 02
1 1 0 1 1
1 1 0 1 1
1 0 1 0 0 0 1 02
Проверка: 110112 =27 ; 1102=6 ; 101000102 = 162; 27×6=162.
Выполнить действия: (11011)2 × (110)2= (10100010)2
(33) 8 × (6)8= (242)8 (1В) 16 × (6)16= (А2)16
Copyright ОАО «ЦКБ «БИБКОМ» & ООО «Aгентство Kнига-Cервис»
8
Деление
Деление осуществляется по тому же алгоритму, что и в десятичной
системе – «деление уголком», также можно воспользоваться таблицей
умножения. От делимого выделяется часть большая делителя, но не больше
чем в t раз (t – основание системы счисления). В результате подбирается
цифра, произведение делителя на которую даст число, меньшее
выделенного, произведение записывается под делимым, сносится
следующая цифра, если получившееся значение числа превосходит
делитель, пишется – подбирается новая цифра, если нет – в частном пишется
0 и сносится следующая цифра и т.д., до получения результата или
достижения требуемой точности.
При проведении арифметических операций над числами,
выраженными в различных системах счисления, необходимо
предварительно перевести их в одну и ту же систему.
Пример 7: Найти частное чисел 10000012 и 1012.
Решение.
Проверка: 10000012 =65; 1012= 5; 11012 = 13; 65: 5= 13.
Выполнить действия:
(10100010)2 : (110)2= (11011)2 (242)8 : (6)8= (33)8 (А2)16 : (6)16=(1В) 16
Задания для самостоятельного выполнения
Задание 1. Перевести в десятичную систему:
1001012 =____________ 10
131,58 =______________10
A0C416 =_____________10
2031,024 =____________10
Задание 2. Перевести в различные системы счисления из десятичной:
93310 =_________________16
45,8310 =________________8
68810 =_____________________8
27,7210 =____________________2
1 0 0 0’ 0 0 12 1 0 12
1 0 1 1 1 0 12
1 1 0
1 0 1
1 0 1
1 0 1
0
Copyright ОАО «ЦКБ «БИБКОМ» & ООО «Aгентство Kнига-Cервис»
9
29510 =_____________________16
83,0310 =____________________4
Задание 3. Выполнить действия, проверить путем перевода в
десятичную систему счисления:
100112-1102=_____________________2
100112+101112 =__________________2
111012×1012 =____________________2
1110002/11102 =___________________2
10001112-101102 =_________________2
100112×101112 =___________________2
10000112+10012 =__________________2
110012/1012 =______________________2
Задание 4. Записать в различных системах счисления, используя
таблицы:
1001001,1102 =_________________4
10011,101112 =_________________16
11101,1012 =___________________8
543,138 =___________________2
15A,C316 =________________________2
416,1138 =___________________2
232,0014 =__________________2
7C1,5916 =______________________2
Вопросы для самоконтроля
1. В чем отличие позиционной системы счисления от
непозиционной?
2. Приведите примеры непозиционных систем счисления.
3. Составьте алгоритм выполнения сложения в двоичной системе
счисления.
4. Составьте алгоритм выполнения вычитания в двоичной системе
счисления.
5. Составьте алгоритм выполнения умножения в двоичной системе
счисления.
6. Составьте алгоритм выполнения деления в двоичной системе
счисления.
Copyright ОАО «ЦКБ «БИБКОМ» & ООО «Aгентство Kнига-Cервис»
10
Индивидуальные задания
1. Перевести в десятичную систему: 1001012 1011,112 0,11012
Перевести числа в двоичную, восьмеричную и шестнадцатеричную
систему счисления из десятичной с точностью до 8 знаков после запятой для
бесконечных дробей: 3310 45,8310 0,12510
Выполнить действия, проверить путем перевода в десятичную:
101110012-1000112 ; 100112+101112;
100112×10112 ; 100001110002/11112.
2. Перевести в десятичную систему: 1101112 1001,012 0,1012
Перевести числа в двоичную, восьмеричную и шестнадцатеричную
систему счисления из десятичной с точностью до 8 знаков после запятой для
бесконечных дробей: 6810 0,7510 27,7210
Выполнить действия, проверить путем перевода в десятичную:
10001010112-111012 ; 100112×101112;
10110112+101111012 ; 101001000002/100002
3. Перевести в десятичную систему: 101101102 1011,1012 0,111012
Перевести числа в двоичную, восьмеричную и шестнадцатеричную
систему счисления из десятичной с точностью до 8 знаков после запятой для
бесконечных дробей: 62910 0,2510 19,3910
Выполнить действия, проверить путем перевода в десятичную:
1011000012-10102 ; 111012+101012;
1011101102×1000102 ; 100001110002/111100.
4. Перевести в десятичную систему: 1101002 101001,12 0,11012
Перевести числа в двоичную, восьмеричную и шестнадцатеричную
систему счисления из десятичной с точностью 8 знаков после запятой для
бесконечных дробей: 29510 0,62510 83,0310
Выполнить действия, проверить путем перевода в десятичную:
10011000012-1011112; 1001012+101112;
101102×10112; 1010002/10002
Copyright ОАО «ЦКБ «БИБКОМ» & ООО «Aгентство Kнига-Cервис»
11
Основные математические структуры
Основные умения: применять основные математические структуры
и арифметические операции над ними к решению профессиональных задач.
Рекомендуемая литература: [7, 12, 15, 18].
Множества
Понятие множества, подмножества, свойства и операции над
множествами см. в части 1 «Лекции» стр. 18-21.
Пример 1: В классе 12 учеников, которые хорошо играют в волейбол,
и 9 учеников, которые хорошо играют в теннис, всего в классе 16 человек.
Сколько учеников хорошо играют и в волейбол, и в теннис?
Решение.
Количество волейболистов V=12;
количество теннисистов Т=9; количество
волейболистов и теннисистов 16=∪ TV .
Найти количество учеников, хорошо
играющих и в волейбол и в теннис
?−∩ TV
.57416
;7916)(
;41216)(|
);()()(
=−−≡∩
=−≡∪=
=−≡∪=
∪=∩
TV
TTVTV
VTVVT
VTTVTVTV
Пример 2: Из 100 студентов экзамен по педагогике сдали 28 человек,
математике – 30 человек, философии – 42 человека, педагогику и
математику сдали 8 человек, математику и философию – 5, педагогику и
философию – 10, все три экзамена – 3 человека. Сколько студентов не сдали
ни одного экзамена?
Решение.
Количество студентов, сдавших
педагогику Р=28; количество студентов,
сдавших математику М=30; количество
человек, сдавших философию F=42;
количество человек, сдавших педагогику
и математику 8;P M∩ = количество
человек, сдавших математику и
философию ;5=∩ FM количество
12 9
?
28 308
42
10 5
3
Copyright ОАО «ЦКБ «БИБКОМ» & ООО «Aгентство Kнига-Cервис»
12
человек, сдавших педагогику и философию ;10=∩ FP количество человек,
сдавших педагогику, математику и философию ;3=∩∩ FMP всего
человек U=100. Найти количество человек, не сдавших ни одного экзамена:
))())(())(())((
)()()((
FMPMFPPFMFMP
PMFFPMFMPU
∩∩∪∩∪∩∪∩
∪∪∪
.
Студенты, сдавшие 3 экзамена: 3.P M F∩ ∩ =
Студенты, сдавшие 2 экзамена:
педагогику и математику: ;538)( =−≡∩ FMP
математику и философию: 235)( =−≡∩ PFM ;
педагогику и философию: .7310)( =−≡∩ MFP
Студенты, сдавшие 1 экзамен:
педагогику – PMF ≡ 28-5-7-3=13;
математику: MPF ≡ 30-5-2-3=20; философию: FMP ≡ 42-2-7-3=30.
Студенты, не сдавшие ни одного экзамена:
100-(13+20+30+5+2+7+3)=100-80=20.
Пример 3: 35 учеников лингвистической гимназии поехали в тур по
Европе. Каждый из гимназистов владеет хотя бы одним иностранным
языком, 25 учеников знают английский язык, 15 – немецкий, 20 –
французский, 15 гимназистов знают английский и французский, 6 –
немецкий и французский, 10 – немецкий и английский языки. Сколько
учеников знают: а) все языки б) только немецкий и французский, но не
знают английского?
Решение:
Языки Английский Французский Немецкий
Английский X U V
Французский Y T
Немецкий Z
X – знают только английский; Y – знают только французский;
Z – знают только немецкий;
U – знают только английский и французский; T – знают только
французский и немецкий; V – знают только английский и немецкий;
W – множество учеников, знающих все 3 языка;
Всего учеников – 35 человек: X+Y+Z+U+V+T+W=35
X+U+V+W=25; Y+U+T+W=20; Z+V+T=W=15
U+W=15 => U=15-W
T+W=6 => T=6-W
V+W=10 => V=10-W
X+15-W+10-W+W=25 => X+25-W=25 => X=W
Copyright ОАО «ЦКБ «БИБКОМ» & ООО «Aгентство Kнига-Cервис»
13
Y+15-W+6-W+W=20 => Y+21-W=20 => Y=W-1
Z+10-W+6-W+W=15 => Z=W-1
W+W-1+W-1+15-W+6-W+10-W+W=35 => W=6
U=9; T=0; V=4; X=6; Y=5; Z=5
Языки Английский Французский Немецкий
Английский 6 9 4
Французский 5 0
Немецкий 5
Ответ: 6 учеников знают все три языка; 10 учеников знают немецкий
или французский и не знают английского
Задания для самостоятельного выполнения
Задание 1. Из группы студентов на занятия физкультурой ходят 20
человек, а в секции – 18 человек, причем 15 человек одновременно ходят и в
секции и на занятия по физкультуре. Сколько студентов освобождены от
занятий спортом, если всего в группе 25 человек?
Задание 2. Группа учащихся из 100 человек собралась в тур по
Европе. 70 человек знают английский язык, 45 знают французский язык и 23
человека знают оба языка. Сколько человек в группе не знают ни
английского, ни французского языка?
Задание 3. Из 40 учеников 30 умеют играть в шашки, 27 умеют играть
в шахматы и только пятеро не умеют ни того, ни другого. Сколько учеников
умеют играть и в шашки и в шахматы?
Задание 4. 20 мальчиков поехали на пикник. При этом 5 из них
обгорели, 8 были сильно покусаны комарами, а 10 остались всем довольны.
Сколько обгоревших мальчиков не было покусано комарами? Сколько
покусанных комарами мальчиков также и обгорели?
Задание 5. В библиотеке вуза читатели обычно берут либо один
учебник по педагогике, либо один задачник по математике, либо один
учебник и один задачник. В один из дней было выдано 57 учебников и 36
задачников. Сколько было читателей в библиотеке, если 12 человек взяли и
учебник и задачник?
Вопросы для самоконтроля
1. Приведите примеры множеств в предметной области Педагогика?
2. Приведите пример подмножества из множества всех студентов
факультета психологии?
3. Назовите свойство, которым связаны элементы множества.
4. Какое множество называется универсальным?
5. Какие операции над множествами существуют?
6. Какое множество называют пустым?
7. Приведите пример бесконечного множества.
Copyright ОАО «ЦКБ «БИБКОМ» & ООО «Aгентство Kнига-Cервис»
14
Индивидуальные задания
1. Числовые множества представлены в виде промежутков. Найдите
и изобразите полученные промежутки с помощью графического редактора
Paint: [8; 15] ∩ [9; 20]; (-1; 1) ∩ [-1; 0); [-1; 1] ∩ [-1; 0]; (-1; 0] ∩ [1; +∞);
[-1; 0) ∪ [0; 4]; [1; +∞) ∪ [0; +∞); {4}∪ (-∞; 4); (0; 2) ∪ [0; 2].
2. На учебе студенческого актива в «Алексин-Боре»
фотокорреспондент университетской газеты взял 2 пленки в 36 и 24 кадра.
После печати фотографий выяснилось, что фотографий со студентами ФФК
получилось – 40, со студентами ФИСиГН – 20, со студентами исторического
факультета – 15. В числе которых было 10 фотографий на которых
запечатлены студенты ФФК и ФИСиГН, 10 – ФИиП и ФФК, 5 – ФИСиГН и
ФИиП и 1 общая фотография. Сколько фотографий получилось, где
запечатлены только студенты ФФК, только ФИСиГН и только
исторического факультета? Сколько фотографий не получилось?
3. Составьте множество двухзначных чисел, в записи которых
используются лишь цифры 2, 5 и 8. Найдите пересечение этого множества с
множеством четных чисел.
4. В 11 «А» классе готовились к выпускному вечеру. Всего в классе
32 человека, из них 17 учащихся написали стихи, 19 – рассказ о школе.
Сколько учеников сочинили и стих, и рассказ?
5. Из класса в 20 человек каждый ходит на какую-нибудь секцию, 15
человек – в спортивную, 10 – в танцевальную и 11 – занимаются хоровым
пением. Причем 4 человека посещают танцы и хоровое пение, 7 –
спортивную и танцевальную секцию, 8 человек посещают хоровое пение и
спортивную секцию. Составить таблицу распределения посещения
различных секций учениками класса.
Copyright ОАО «ЦКБ «БИБКОМ» & ООО «Aгентство Kнига-Cервис»
15
Комбинаторика
Основные формулы комбинаторики более подробно разобраны в
части «Тезисы лекций» стр. 22-24.
Правило суммы
Пусть имеется n попарно непересекающихся множеств A1, A2,…,An
содержащих m1, m2,…,mn элементов соответственно. Число способов,
которыми можно выбрать один элемент из всех этих множеств, равно
m1 + m2 +…+ mn
Пример 1: На курсе имеется 3 группы. В первой – 25 человек, во
второй – 30, в третьей – 20. Сколькими способами из них можно выбрать
одного студента?
Правило произведения
Пусть имеется n множеств A1, A2,…,An, содержащих m1, m2,…,mn
элементов соответственно. Число способов, которыми можно выбрать по
одному элементу из каждого множества, то есть построить кортеж (а1, а2, …,
аn), где аi ∈ Аi, i=1,2,…,n, равно m1ּ m2 ּ …ּmn.
Пример 2: На курсе имеется 3 группы. В первой – 25 человек, во
второй – 30, в третьей – 20. Сколькими способами из каждой из них можно
выбрать по одному студенту?
Перестановки без повторений
Различные кортежи, которые можно построить из элементов данного
множества, взятых ровно по одному разу, называются перестановками.
!nPn =
Факториал (n!) – это последовательное произведение натуральных
чисел от 1 до n: n!=1⋅2⋅…⋅ n.
Пример 3: Сколькими способами можно расставить в шеренгу
студентов группы из 25 человек?
Размещения без повторений
Различные упорядоченные подмножества по m элементов данного
множества, содержащего n элементов, называются размещениями из n по m.
( )!
!
mn
n
Am
n
−
=
Пример 4: Из класса, состоящего из 25 учащихся, необходимо
выбрать команду из четырех человек на шахматный турнир на I, II, III и IV
доски. Сколькими способами это можно сделать?
Copyright ОАО «ЦКБ «БИБКОМ» & ООО «Aгентство Kнига-Cервис»
16
Сочетания без повторений
Различные неупорядоченные подмножества по m элементов из
данного множества, содержащего n элементов, называются сочетаниями из n
по m.
( )!!
!
mnm
n
C m
n
−
=
В частности, С 1
n = n; С 0
n = 1, С n
n = 1.
Пример 5: Сколькими способами из группы в 25 человек можно
выбрать команду из пяти человек на олимпиаду по математике?
Размещения с повторениями
Различные кортежи длины m, составленные из элементов данного
множества, содержащего n элементов, так, что эти элементы в кортеже могут
повторяться, называются размещениями с повторениями из n по m.
mm
n nA =
~
Пример 6: Группа студентов педагогического вуза, состоящая из 25
человек сдает экзамен. Возможные оценки – 2, 3, 4, 5. Сколькими способами
может быть заполнена экзаменационная ведомость?
Перестановки с повторениями
Составом кортежа длины n, состоящего из элементов a1,a2,…,ak,
называется кортеж (n1, n2,…,nk), где ni – число вхождений элемента аi, i=1,
2,…,k, в данный кортеж, причем n1+n2+…+nk=n. Перестановки с
повторениями – это различные кортежи данного состава.
!!!
!
),...,,(
21
21
k
kn
nnn
n
nnnP
K⋅
=
Пример 7: Сколько различных «слов» можно составить, переставляя
буквы слова «математика»?
Сочетания с повторениями
Различные неупорядоченные наборы, составленные из m элементов
данного множества так, что элементы в наборе могут повторяться, и порядок
их не важен, называются сочетаниями с повторениями из n по m.
m
nС
~
= )!1n(!m
)!1mn(
−
−+
Пример 8: Группа студентов педагогического вуза, состоящая из 25
человек, сдала экзамен. В отчете показано, сколько каких оценок было
получено. Сколькими способами может быть составлен отчет?
Copyright ОАО «ЦКБ «БИБКОМ» & ООО «Aгентство Kнига-Cервис»
17
Алгоритм решения комбинаторных задач
При решении комбинаторных задач следует ответить на следующие
вопросы:
1. Из какого множества осуществляется выбор (найти n –
количество элементов из которых составляются комбинации)?
2. Сколько элементов в одной комбинации (найти m; если n=m, то
применяем правило перестановок P и переходим к вопросу 4)?
3. Важен ли порядок (изменится ли комбинация, если в ней
поменять элементы местами)?
если важен, применяем правило размещений А,
если нет – сочетаний С.
4. Возможны ли повторения элементов в одной комбинации?
Рекомендуемая литература: [7, 12, 15, 18].
Пример 9: В фортепианном кружке дома детского творчества
занимается 10 человек, в кружке художественного слова – 15, в вокальном –
12 и в фотокружке – 20. Сколькими способами можно составить команду из
4 чтецов, 3 пианистов, 5 певцов и одного фотографа для выезда на
экскурсию?
Решение. Разобьем решение задачи на подзадачи.
1. Сначала найдем сколькими способами можно выбрать чтецов.
1) производим выбор из 15 человек, n=15;
2) выбираем 4 человек, m=4;
3) порядок не важен, т.е. используем правило сочетаний С 4
15;
4) сочетания без повторений, так как люди выбираются разные.
2. Проводя подобные рассуждения, выбираем пианистов:
3 из 10 – С 3
10 способов.
3. Певцов: 5 из 12 – С 5
12 способов.
4. Фотографа: 1 из 20 – С 1
20 способов.
Поскольку выбор производится по всем четырем позициям, а не по
одной, применяем правило произведения:
91
20
5
12
3
10
4
15 10595,220
)!512(!5
!12
)!310(!3
!10
)!415(!4
!15
⋅=⋅
−
⋅
−
⋅
−
=⋅⋅⋅ CCCC
Ответ: команду можно составить 2,595ּ109
способами.
Пример 10: В классе 15 мальчиков и 20 девочек. Для концерта надо
выделить танцевальный дуэт, дуэт певцов, дуэт гимнастов. Сколькими
способами это можно сделать при условии, что все дети могут танцевать,
петь, выполнять гимнастические упражнения, а дуэт должны составлять
мальчик и девочка?
Copyright ОАО «ЦКБ «БИБКОМ» & ООО «Aгентство Kнига-Cервис»
18
Решение. Разобьем задачу на подзадачи.
1. Найдем, сколькими способами для концерта можно выбрать
мальчиков.
1) n=15;
2) m=3;
3) порядок важен, так как в списке фамилии будут записаны по
порядку, кто – гимнаст, кто – певец и т.д. То есть применим правило
размещений А 3
15 ;
4) повторения невозможны, так как одного человека нельзя
выбрать несколько раз.
2. Найдем сколькими способами для концерта можно выбрать
девочек.
1) n=20;
2) m=3;
3) порядок важен;
4) повторения невозможны.
Для того чтобы найти окончательный ответ, перемножим полученные
выражения (применяем правило произведения потому, что выбираются и
мальчик, и девочка):
А 3
15 ּ А 3
20 =2730 ּ6840=18673200
Ответ: три дуэта можно выбрать 18673200 способами.
Задания для самостоятельного выполнения
Задание 1. Сколькими способами 5 человек могут построиться в
шеренгу?
n=____ m=_____ Порядок важен?______ Повторения __________
Формула: __________________ Ответ: ___________________
Задание 2. Сколькими способами из группы детей начального
школьного возраста в 18 человек можно выбрать двоих участников детского
утренника?
n=____ m=_____ Порядок важен?______ Повторения __________
Формула: __________________ Ответ: ___________________
Задание 3. Сколькими способами из группы студентов 18 человек
можно выбрать актив группы: старосту, зам. старосты, физорга и затейника.
n=____ m=_____ Порядок важен?______ Повторения __________
Формула: __________________ Ответ: ___________________
Copyright ОАО «ЦКБ «БИБКОМ» & ООО «Aгентство Kнига-Cервис»
19
Задание 4. Сколько шестибуквенных слов можно составить из
карточек, из которых составлено слово «ананас»?
n=____ m=_____ Порядок важен?______ Повторения __________
Формула: __________________ Ответ: ___________________
Задание 5. Сколько существует различных пятизначных телефонных
номеров, если первая цифра номера всегда 2?
Задание 6. Сколькими способами можно расставить 12 черных и 12
белых шашек на 32 черных клетках шахматной доски?
Вопросы для самоконтроля
1. Какие задачи называются комбинаторными?
2. Сформулируйте правило произведения.
3. Сформулируйте правило суммы.
4. Расскажите алгоритм решения комбинаторных задач.
Индивидуальные задания
1. В библиотеке на полке стоят 5 книг. Сколько существует
различных способов их расположения?
2. Сколькими способами в команде КВН, состоящей из 10 человек,
можно выбрать капитана и ведущего?
3. Сколькими способами из группы спортсменов в 12 человек
можно выбрать двоих участников соревнования?
4. Сколько различных слов (не обязательно осмысленных) можно
получить перестановками всех букв в слове РОТОР, ТОПОР?
5. Для школы учителю необходимо купить тетради в линейку. В
магазине продаются 7 различных видов тетрадок. Сколько существует
способов покупки 4 тетрадок.
6. В третьем классе 13 мальчиков и 12 девочек. Для концерта
необходимо создать три дуэта, в каждом из них должен быть один мальчик и
одна девочка. Сколькими способами это можно сделать?
7. 7 девочек водят хоровод. Сколькими различными способами они
могут встать в круг?
8. Сколько трехзначных чисел можно составить из цифр 1,2,3,4,0,
если цифры в числе могут повторяться?
9. На урок физкультуры в 1 классе пришли 5 учеников. На первом
занятии учитель обучает их, как рассчитаться по порядку. По команде «По
порядку номеров рассчитайся» учащиеся производят расчет: «первый –
второй – третий – четвертый – пятый». После этого учитель перестраивает их
Copyright ОАО «ЦКБ «БИБКОМ» & ООО «Aгентство Kнига-Cервис»
20
по-новому и расчет повторяется. Сколько раз может учитель физкультуры
повторить это упражнение, используя только разные способы построения
первоклассников?
10. Сколькими способами можно расселить 8 студентов в одно-, трех-
и четырехместной комнатах?
11. Сколькими способами из группы в 24 человека можно выбрать
двоих делегатов на конференцию.
12. В почтовом отделении продаются открытки 10 видов. Сколькими
способами можно купить 12 открыток?
13. Телефонные номера в городе состоят из шести цифр. Сколько
всего телефонных номеров может быть в городе, если номер начинается
только на цифру 5?
14. На почте имеются марки 5 различных типов. Сколько существует
способов покупки 10 марок?
15. Сколькими различными способами могут разместиться на
скамейке 4 человека?
16. Сколько различных шестизначных чисел можно записать с
помощью цифр: 1,1,1,2,2,2?
17. Однажды утром по улице города на высокой скорости пронеслась
машина. Она сбила зазевавшегося поросенка и скрылась в неизвестном
направлении. Возвращавшийся из ресторана житель города заметил номер
автомобиля. Но когда появилась милиция, он с перепугу вспомнил буквы
номера и, что все цифры номера нечетные и различные, но точно не помнил
их расположения. Сколько номеров нужно проверить по картотеке ГИБДД,
чтобы найти нарушителя, если в номере 3 цифры?
Copyright ОАО «ЦКБ «БИБКОМ» & ООО «Aгентство Kнига-Cервис»
21
Определения и свойства вероятности
Основные умения: решать задачи на определение вероятности;
научиться определять вероятность суммы и произведения событий;
выделять гипотезы и применять к решению задач формулы полной
вероятности и Бейеса.
Рекомендуемая литература: [7, 8, 12, 15, 18].
Определения вероятности
Наблюдение явления, опыт, эксперимент, которые можно провести
многократно, в теории вероятностей принято называть испытанием.
Результат, исход испытания, называется событием.
Событие называется случайным, если его наступление или
ненаступление в некотором испытании (эксперименте) зависит от ряда
случайных факторов.
Понятие о случайном событии см. в части 1 «Лекции» стр. 25.
Совокупность образует полную группу событий для данного
испытания, если его результатом обязательно становится хотя бы одно из
них.
События, образующие полную группу попарно несовместных и
равновозможных событий, будем называть элементарными событиями.
Классическое определение вероятности
Вероятностью P(A) события A называется отношение числа
элементарных событий m, благоприятствующих событию A, к числу всех
элементарных событий n:
n
m
AP =)(
Пример 1: В урне 20 шаров: 15 белых и 5 черных. Какова вероятность
извлечь черный шар, если вынимается один шар?
Решение. Пусть событие А – вынуть черный шар, тогда
m = 5 – количество черных шаров, n = 20 – количество всех шаров.
5 1
( ) 0,25
20 4
m
P A
n
= = = =
Статистическое определение вероятности
Пусть проводятся n опытов событие A наступило m раз,
тогда
n
m
AP =)( , где m – абсолютная частота события A, P(A) –
относительная частота события A.
Copyright ОАО «ЦКБ «БИБКОМ» & ООО «Aгентство Kнига-Cервис»
22
Gмера
gмера
AP =)(
Вероятностью события А для испытания в данном опыте называется
число Р(А), около которого группируются значения относительной частоты
при больших n.
Пример 2: Французский естествоиспытатель XVIII в. Ж.Л. Бюффон
при экспериментальной проверке закона больших чисел бросил монету 4 040
раз, в результате чего герб выпал 2 048 раз. Найти относительную частоту
выпадения герба в данном эксперименте.
Решение. Событие А – выпадение герба, абсолютная частота
появления герба m = 2048, общее количество n = 4040, тогда
( ) 51,0
4040
2048
≈==ΑΡ
n
m
Геометрическое определение вероятности
Области могут быть различной размерности
(одно-, двух- или трехмерного измерения), и в
зависимости от выбора размерности меры могут
принимать значения либо длины, либо площади, либо объема. Для
конкретного испытания размерность мер g и G должна быть одна.
Пример 3. В квадрат со стороной 1 см случайным образом бросается
точка, положение которой равновозможно в любом месте квадрата. Какова
вероятность, что расстояние от нее до центра квадрата не больше 0,5 см?
Решение. Область, в которую должна попасть
точка, представляет собой круг с центром в точке
пересечения диагоналей квадрата и радиусом 0,5 см.
Следовательно, S – площадь квадрата, S1 – площадь
круга. Событие А – точка попала в круг. S= 12
см2
.
S1 = πR2
= 3,14 · (0,5)2
=3,14·0,25=0,785 см2
.
Вероятность попадания точки в искомую область
равна:
785,0
1
785,0
)( 1
===
S
S
AP
Задания для самостоятельного выполнения
Задание 1. Из колоды карт в 52 штуки вынимают одну. Какова
вероятность извлечь из колоды любую карту крестовой масти?
A – _____________________________________________________
n=________ m=_______ Формула: _______________Ответ: _______
Copyright ОАО «ЦКБ «БИБКОМ» & ООО «Aгентство Kнига-Cервис»
23
Задание 2. Бросают игральную кость. Найти вероятность того, что на
верхней грани появится четное число очков?
A – _____________________________________________________
n=________ m=_______ Формула: _______________Ответ: _______
Задание 3. В урне 4 белых и 3 черных шара. Наугад вынимают два
шара. Какова вероятность, что они разного цвета?
A – _____________________________________________________
n=________ m=_______ Формула: _______________Ответ: _______
Задание 4. Среди 25 учеников класса, в котором 10 девочек,
разыгрывается 5 билетов в цирк. Найти вероятность того, что среди
обладателей билетов окажется 2 девочки?
A – _____________________________________________________
n=________ m=_______ Формула: _______________Ответ: _______
Вопросы для самоконтроля
1. Что такое частота событий?
2. Что такое вероятность события?
3. Чему равна вероятность достоверного события?
4. Чему равна вероятность невозможного события?
5. В каких пределах заключена вероятность события?
6. Какое определение вероятности называют статистическим?
Индивидуальные задания
1. Все натуральные числа от 1 до 30 записаны на одинаковых
карточках и помещены в урну. После тщательного перемешивания карточек
из урны извлекается одна карточка. Какова вероятность того, что число на
взятой карточке окажется кратным 5?
2. В урне лежат 4 белых и 3 черных шара. Наугад вынимаются
2 шара. Какова вероятность, что они черного цвета?
3. В группе 5 юношей и 3 девушки. Среди них по жребию
разыгрываются два билета в кино. Какова вероятность, что билеты
достанутся юноше и девушке?
4. В круг радиуса 2 случайным образом бросается точка, положение
которой равновозможно в любом месте круга. Какова вероятность, что
расстояние от нее до центра круга не больше 1,5?
Copyright ОАО «ЦКБ «БИБКОМ» & ООО «Aгентство Kнига-Cервис»
24
5. В ящике находятся 15 красных, 9 голубых и 6 зеленых шаров. Из
ящика подряд вынимаются шары без возвращения обратно. Какова
вероятность того, что из 6 вынутых шаров окажется 1 зеленый, 2 голубых и
3 красных шара?
6. Из карточек, из которых составлено слово «дисплей», случайным
образом выбраны три и выложены в ряд. Какова вероятность, что они
образовали слово «лес»?
7. Студент сдает в сессию три экзамена. За каждый экзамен
равновозможны оценки «5», «4», «3», «2». Стипендия назначается при
отсутствии оценок «3» и «2». Какова вероятность назначения стипендии?
8. Участники жеребьевки тянут из ящика жетоны с номерами от 1 до
100. Найти вероятность того, что номер первого наудачу извлеченного
жетона не содержит цифры 5.
9. В куб с ребром 1 случайным образом бросается точка, положение
которой равновозможно в любом месте куба. Какова вероятность, что
расстояние от нее до выделенной вершины куба больше 0,5?
10. На окружности случайным образом выбрали две точки,
положение которых равновозможно в любом месте окружности. Какова
вероятность, что расстояние между ними больше радиуса окружности?
11. На 5 одинаковых карточках написаны буквы Б, Е, Р, С, Т. Эти
карточки наудачу разложили в ряд. Какова вероятность того, что
получилось слово БРЕСТ?
12. В квадрат со стороной 2 случайным образом бросается точка,
положение которой равновозможно в любом месте квадрата. Какова
вероятность, что расстояние от нее до центра квадрата больше 0,5?
13. Среди тысячи новорожденных оказалось 517 мальчиков. Найти
относительную частоту рождения мальчиков.
14. Студент из 20 вопросов к зачету усвоил 16. Билет состоит из двух
вопросов. Какова вероятность, что он не знает ответа ни на один из
доставшихся ему вопросов?
15. В круг радиуса 3 случайным образом бросается точка, положение
которой равновозможно в любом месте круга. Какова вероятность, что
расстояние от нее до центра круга не больше 1?
Copyright ОАО «ЦКБ «БИБКОМ» & ООО «Aгентство Kнига-Cервис»
25
Свойства вероятности, формула полной вероятности,
формула Бейеса
Произведением событий А и В называется событие С=А⋅В, состоящее
в том, что в результате испытания произошло и событие А и событие В, т.е.
оба события произошли.
Два события A и B называются независимыми, если вероятность
появления каждого из них не зависит от того, появилось другое событие или
нет. В противном случае события A и B называются зависимыми.
Теорема 1. Вероятность произведения двух независимых событий
A и B равна произведению этих вероятностей.
P(A⋅B)=P(A) ⋅P(B)
Пусть А и В – зависимые события. Условной вероятностью PA(B)
события В называется вероятность события В, найденная в предположении,
что событие А уже наступило.
Теорема 2. Вероятность произведения двух зависимых событий A и B
равна произведению вероятности одного из них на условную вероятность
другого, найденного в предположении, что первое событие уже наступило.
P(A⋅B)=P(A) ⋅PА (B)
Два события называются совместными, если появление одного из
них не исключает появление другого в одном и том же испытании.
Два события называются противоположными, если в данном
испытании они несовместны и одно из них обязательно происходит.
Суммой событий A и B называется событие C=A+B, состоящее в
наступлении, по крайней мере, одного из событий A или B, т.е. наступлении
события А, или события В, или обоих этих событий вместе, если они
совместны.
Теорема 3. Вероятность суммы двух несовместных событий A и B
равна сумме вероятностей этих событий:
P(A+B)=P(A)+P(B)
Следствие. Сумма вероятностей противоположных событий А и Ā
равна единице:
P(A)+P(Ā)=1
Вероятность суммы полной группы событий равна 1.
Теорема 4. Вероятность суммы двух совместных событий A и B равна
сумме вероятностей этих событий минус вероятность их произведения
P(A+B) = P(A) + P(B) – P(A⋅B)
Теорема 5 (Формула полной вероятности). Вероятность события A,
которое может наступить лишь при условии появления одного из n попарно
Copyright ОАО «ЦКБ «БИБКОМ» & ООО «Aгентство Kнига-Cервис»
26
несовместных событий B1,B2 ,…, Bn образующих полную группу, равна
сумме произведений вероятностей каждого из этих событий на
соответствующую условную вероятность события A:
)()(...)()()()()( 21 21 APBPAPBPAPBPAP nBnBB ⋅++⋅+⋅=
или ∑=
=
n
i
Bi APBPAP i
1
)()()( .
Теорема 6 (Формула Бейеса). Если существуют n попарно
несовместных событий B1,B2,…,Bn образующих полную группу, известны
условные вероятности события А, то можно найти вероятности, того что
событие А произошло при условии появления некоторого события Вk, можно
найти по формуле:
( )
( ) ( )
( ) ( ) ( ) ( )APBPAPBP
APBP
BP
n
k
BnB
Bk
kA
++
⋅
=
...11
Примеры решения задач на вычисление вероятности по формулам
полной вероятности и Бейеса см. в части 1 «Лекции» стр. 31-32.
Пример 1: Вероятность попадания для первого стрелка 0,85, для
второго – 0,8. Спортсмены независимо друг от друга сделали по одному
выстрелу.
1) Найти вероятность того, что в мишень попадут оба стрелка?
Решение.
А – попал 1-ый стрелок
B – попал 2-ой стрелок
События А и В – независимые. Событие А⋅В состоит в том, что в
результате испытания произошло и событие А и событие В, т.е. оба события.
P(A)=0,85
P(B)=0,8.
Формула: P(AB)=P(A) ⋅ P(B).
P(AB)=0,85⋅0,8
Ответ: 0,68
2) Найти вероятность того, что промахнутся оба стрелка
Решение.
À – промахнулся 1-ый стрелок Â – промахнулся 2-ой стрелок
Событие À является противоположным событию А, т.е.
Р( À )=1-P(A)=1-0,85=0,15
Событие Â является противоположным событию В, т.е.
Copyright ОАО «ЦКБ «БИБКОМ» & ООО «Aгентство Kнига-Cервис»
27
Р( Â )=1-P(В)=1-0,8=0,2.
События À и Â независимые.
Формула: P( ÀÂ )=P( À )⋅P( Â ).
P( ÀÂ )=0,15⋅0,2
Ответ: 0,03
3) Найти вероятность того, что в мишень попадет хотя бы один
стрелок
Решение.
А – попал 1-ый стрелок, B – попал 2-ой стрелок
События А и В – совместные. Событие А+В состоит в том, что в
результате испытания произошло или событие А, или событие В, или оба
события.
P(A)=0,85
P(B)=0,8.
Формула: P(A+B) = P(A) + P(B) – P(A⋅B)
P(A+B)=0,85+0,8-0,68
Ответ: 0,97
4) Найти вероятность того, что в мишень попадет ровно один стрелок.
Решение.
А – попал 1-й стрелок; À – промахнулся 1-й стрелок
B – попал 2-й стрелок; Â – промахнулся 2-й стрелок
Событие «в мишень попадет ровно один стрелок» означает, что попал
1-й стрелок и промахнулся 2-й или промахнулся 1-й и попал 2-й стрелок.
P(A)=0,85 Р( À )=1-P(A)=1-0,85=0,15
P(B)=0,8 Р( Â )=1-P(В)=1-0,8=0,2.
Формула: )()()()()( BPAPBPAPBABAP ⋅+⋅=⋅+⋅
=⋅+⋅ )( BABAP 0,85⋅0,2+0,15⋅0,8
Ответ: 0,29
Пример 2: В группе из 20 студентов: 5 – отличников, 10 – хорошо
успевающих и 5 – занимающихся слабо. На предстоящем экзамене
отличники могут получить только отличные оценки. Хорошо успевающие
студенты могут получить с равной вероятностью хорошие и отличные
оценки. Слабо занимающиеся студенты могут получить с равной
вероятностью хорошие, удовлетворительные и неудовлетворительные
оценки. Для сдачи экзамена приглашается наугад один студент. Найти
вероятность того, что он получит хорошую или отличную оценку.
Copyright ОАО «ЦКБ «БИБКОМ» & ООО «Aгентство Kнига-Cервис»
28
Решение.
А – студент получил хорошую или отличную оценку
B1 – приглашен студент – отличник Р(B1)=0,25
B2 – приглашен студент – хорошист Р(B2)=0,5
B3 – приглашен слабый студент Р(B3)=0,25
Формула:
)()()()()()()( 321 321 APBPAPBPAPBPAP BBB ⋅+⋅+⋅=
Найдем условные вероятности ),(1
APB ),(2
APB )(3
APB :
1)(1
=APB 1)(2
=APB
3
1
)(3
=APB
В соответствии с формулой полной вероятности получаем:
Р(А) = 0,25 ⋅1 + 0,5 ⋅ 1 + 0,25 ⋅1/3
Ответ: 0,8333
Пример 3: В пяти ящиках находятся одинаковые по весу и размерам
шары. В двух ящиках – по 6 голубых и 4 красных шара (это ящик состава
B1). В двух других ящиках (состав B2) – по 8 голубых и 2 красных шара. В
одном ящике (состав B3) – 2 голубых и 8 красных шаров. Наудачу
выбирается ящик и из него извлекается шар. Извлеченный шар оказался
голубым. Какова вероятность того, что голубой шар извлечен из ящика
первого состава?
Решение.
А – из ящика извлечен голубой шар
B1 – шар извлечен из ящика первого состава 4,0
5
2
)( 1 ==BP
B2 – шар извлечен из ящика второго состава 4,0
5
2
)( 2 ==BP
B3 – шар извлечен из ящика третьего состава 2,0
5
1
)( 3 ==BP
Формула:
( )
( ) ( )
( ) ( ) ( ) ( ) ( ) ( )APBPAPBPAPBP
APBP
BP
BBB
B
A
321
1
321
1
1
++
⋅
=
Найдем условные вероятности ),(1
APB
),(2
APB )(3
APB :
6,0
10
6
)(1
==APB 8,0
10
8
)(2
==APB 2,0
10
2
)(3
==APB
Copyright ОАО «ЦКБ «БИБКОМ» & ООО «Aгентство Kнига-Cервис»
29
В соответствии с формулой Бейеса найдем искомую вероятность:
( )
2,02,08,04,06,04,0
6,04,0
1
⋅+⋅+⋅
⋅
=BPA
Ответ: 0,4
Задания для самостоятельного выполнения
Задание 1. В урне 40 шаров: 15 голубых, 5 зеленых и 20 белых.
Какова вероятность того, что из урны будет извлечен цветной шар?
A – __________________________________________
B – __________________________________________
А⋅ В
зависимые независимые
А+В
совместные несовместные
Формула: ________________________________
Ответ: _____________
Задание 2. В шкатулке 6 синих и 4 красных катушки ниток. Из нее
извлекают подряд две катушки. Какова вероятность, что обе вынутые
катушки с синими нитками?
A – __________________________________________
B – __________________________________________
А⋅ В зависимые независимые
А+В совместные несовместные
Формула: _________________________________
Ответ:______________
Задание 3. Найти вероятность того, что наудачу взятое двухзначное
число окажется кратным 2 или 7.
Задание 4. В урне 15 шаров, из которых 5 голубых и 10 красных. Из
нее последовательно вытаскивают два шара. Какова вероятность, что
первым извлеченным окажется красный шар, а вторым – голубой?
Задание 5. Студент знает 20 из 25 вопросов программы. Определить
вероятность того, что
1) студент знает все три предложенных ему вопроса;
2) студент не знает ни один из трех предложенных ему вопроса.
Задание 6. Для приема экзамена заготовлено 50 задач. 20 задач из
раздела «А», 20 – «В», 10 – «С». Для сдачи экзамена студент должен решить
первую попавшуюся задачу. Какова вероятность сдать экзамен, если он
умеет решать все задачи из раздела «А», 10 из «В» и 5 из «С»? Какова
вероятность, что ему попалась задача из раздела «А»?
Copyright ОАО «ЦКБ «БИБКОМ» & ООО «Aгентство Kнига-Cервис»
30
Вопросы для самоконтроля
1. Чему равна вероятность суммы двух совместных событий?
2. Чему равна вероятность суммы двух несовместных событий?
3. Чему равна сумма вероятностей противоположных событий?
4. Как определить вероятность произведения двух независимых
событий?
5. Как связаны между собой формула Бейеса и формула полной
вероятности?
6. В каких случаях применяют формулу Бейеса?
7. В каких случаях применяют формулу полной вероятности?
Индивидуальные задания
1. Бросается один раз игральная кость. Определите вероятность
того, что выпадет 1 или 6 очков.
2. В школу поступили партии книг, выпущенные двумя
издательствами: 25% – из первого издательства и 75% со второго. В
продукции первого издательства 60% учебников по математике, второго –
80%. Какова вероятность, что наудачу взятая книга окажется учебником по
математике?
3. В группе спортсменов 20 лыжников и 8 бегунов и 7 пловцов.
Вероятность выполнить квалификационную норму для лыжника равна 0,8,
для бегуна – 0,7 для пловца – 0,85. Спортсмен выполнил норму. Какова
вероятность того, что он лыжник?
4. В лабиринте на правильном маршруте имеются три развилки, на
каждой из которых нужно выбирать одно из двух направлений. Какова
вероятность, что испытуемый пройдет лабиринт с первой попытки?
5. Из 50 табуреток 18 изготовлены учениками 7 «А» класса, 20 –
учениками 7 «Б», остальные – в 7 «В». Ученики 7 «Б» и 7 «В» обычно
выполняют качественную продукцию с вероятностью 0,9. В продукции
учеников 7 «А» бракованные изделия составляют 15%. Взятая наудачу
табуретка оказалась качественной. Какова вероятность того, что табуретка
изготовлена учениками 7 «Б» класса?
6. Какова вероятность, что при троекратном бросании монеты
выпали только орлы?
7. Учебное заведение получает канцелярские товары от трех
магазинов: 1– 30 %; 2 – 55 %; 3 – 15 %. Известно, что брак составляет: 1 –
5%; 2 – 6%; 3 – 10%. Полученные канцелярские товары хранятся на общем
складе. Какова вероятность того, что наугад взятый товар оказался
бракованным?
Copyright ОАО «ЦКБ «БИБКОМ» & ООО «Aгентство Kнига-Cервис»
31
Случайные величины
Основные умения: находить характеристики случайных величин;
применять законы распределения случайных величин к решению
профессиональных задач.
Рекомендуемая литература: [7, 12, 15].
Понятие случайной величины
Случайной величиной (СВ) называется переменная величина,
которая в зависимости от исхода испытания случайно принимает одно
значение из множества возможных, зависящих от случая.
Случайная величина, принимающая различные значения, которые
можно записать в виде конечной или бесконечной последовательности,
называется дискретной случайной величиной.
Случайная величина, которая может принимать все значения из
некоторого промежутка, называется непрерывной случайной величиной.
Представление о случайных событиях, результатом которых
являются числовые значения см. в части 1 «Лекции» стр. 34.
Закон распределения случайных величин
Законом распределения дискретной СВ называется соответствие
между значения x1, x2, …, xn этой величины и их вероятностями р1, р2, … рn.
Закон распределения дискретной СВ может быть задан таблично или
аналитически (то есть с помощью формул).
Х х1 х2 … хn-1 хn
Р р1 р2 … рn-1 рn
где x1, x2, …, xn – случайные величины, соответствующие полной группе
событий: р1+р2+…+ рn = 1.
Математическим ожиданием М(Х) дискретной случайной величины
Х называется сумма произведений всех возможных значений величины Х на
соответствующие вероятности:
∑ +++==
=
n
i
nnii
pxpxpxpxXM
1
2211
...)(
.
Дисперсией D(X) дискретной случайной величины Х называется
математическое ожидание квадрата отклонения случайной величины Х от ее
математического ожидания: D(X) = M [(X – M(X))2
]. Вычисления удобней
производить по формуле: D(X) = M(X2
) – M2
(X).
Copyright ОАО «ЦКБ «БИБКОМ» & ООО «Aгентство Kнига-Cервис»
32
Средним квадратическим отклонением σ(Х) случайной величины Х
называется корень квадратный из ее дисперсии: )()( XDX =σ .
Возможности применения законов распределения случайных величин и
их характеристик к исследованию процессов реального мира представлены
в части 1 «Лекции» стр. 36-40.
Биномиальное распределение
Пусть производится n испытаний, причем вероятность появления
события А в каждом испытании равна р, то вероятность его ненаступления
равна q = 1 – р. Найдем вероятность того, что при n испытаниях событие А
наступит т раз (m ≤ n).
Формула Бернулли: ( )
( )
mnm
n qp
n-mm
n
mP −
=
!!
!
Закон биномиального распределения
Х 0 1 … m … n
р qn
npqn-1
Cn
m
pm
qn-m
pn
Пример 1: Пусть всхожесть семян определенного растения составляет
90 %. Найти вероятность того, что из четырех посеянных семян взойдут: а)
три; б) не менее трех. Напишите в виде таблицы закон распределения
случайной величины Х – всхожесть семян.
Решение. а) в данном случае n = 4; m = 3; p = 0,9; q = 1 – p; q = 0,1.
Применяя формулу Бернулли, получим:
( )
( )
( ) ;2916,01,09,0
1!!3
!4
!!
! 3
==⋅= −mnm
n qp
n-mm
n
mP
б) искомое событие А состоит в том, что из четырех семян взойдут
или три, или четыре. По теореме сложения вероятностей получим:
;(4)(3))( 44 PPAP += ( ) ;6561,09,0(4)
4
4
==P
6561,02916,0)( +=AP =0,9477
Закон распределения:
n = 4; p = 0,9; q = 0,1:
для m = 0 (семена не прорастут) P4(0)=qn
=(0,1)4
=0,0001;
для m = 1 (прорастет 1 семя) P4(1)=npqn-1
=4⋅0,9⋅(0,1)3
=0,0036;
для m = 2 P4(2)=С4
2
pqn-1
=6⋅(0,9)2
⋅(0,1)2
=6⋅0,81⋅0,01=0,0486;
для m = 3 P4(3)= 0,2916;
для m = 4 (прорастут все семена) P4(4)=0,6561.
Copyright ОАО «ЦКБ «БИБКОМ» & ООО «Aгентство Kнига-Cервис»
33
Х 0 1 2 3 4
P 0,0001 0,0036 0,0486 0,2916 0,6561
Проверка: 0,0001+0,0036+0,0486+0,2916+0,6561=1.
В MS Excel функция БИНОМРАСП применяется для вычисления
вероятности в задачах с фиксированным числом тестов или испытаний,
когда результатом любого испытания может быть только успех или неудача.
Функция использует следующие параметры:
БИНОМРАСП (число_успехов; число_испытаний;
вероятностъ_успеха; интегральная), где
число_успехов – количество успешных испытаний;
число_испытаний – число независимых испытаний (число успехов и
число испытаний должны быть целыми числами);
вероятность_ успеха – вероятность успеха каждого испытания;
интегральная – логическое значение, определяющее вид функции:
если параметр имеет значение ИСТИНА (=1), то считается
интегральная функция распределения (вероятность того, что число
успешных испытаний не менее значения число_ успехов);
если параметр имеет значение ЛОЖЬ (=0), то вычисляется значение
функции плотности распределения (вероятность того, что число успешных
испытаний в точности равно значению аргумента число_ успехов).
Пример 2: Какова вероятность того, что
а) трое из четырех новорожденных будут мальчиками?
б) не более трех из четырех новорожденных будут мальчиками?
Решение.
1. Устанавливаем табличный курсор в свободную ячейку, например в
А1. Здесь должно оказаться значение искомой вероятности.
2. Для получения значения вероятности воспользуемся функцией:
нажимаем на панели инструментов кнопку Вставка функции fx.
3. В появившемся диалоговом окне Мастер функций – шаг 1 из 2
слева в поле Категория указаны виды функций. Выбираем
Статистическая. Справа в поле Функция выбираем функцию
БИНОМРАСП и нажимаем на кнопку ОК.
Появляется диалоговое окно функции. В поле Число_s вводим с
клавиатуры количество успешных испытаний (3). В поле Испытания вво-
дим с клавиатуры общее количество испытаний (4). В рабочее поле
Вероятность_s вводим с клавиатуры вероятность успеха в отдельном
испытании (0,5). В поле Интегральная вводим с клавиатуры вид функции
распределения – интегральная (0). Нажимаем на кнопку ОК.
а) в ячейке А1 появляется искомое значение вероятности р = 0,25.
Ровно 3 мальчика из 4 новорожденных могут появиться с вероятностью 0,25.
Copyright ОАО «ЦКБ «БИБКОМ» & ООО «Aгентство Kнига-Cервис»
34
б) Если выяснить вероятность того, что появится не менее трех
мальчиков, то в этом случае в рабочее поле Интегральная вводим 1 (вид
функции распределения интегральная). Вероятность этого события будет
равна 0,9375.
Нормальное распределение
Нормальное распределение – это совокупность объектов, в которой
крайние значения некоторого признака – наименьшее и наибольшее – появ-
ляются редко; чем ближе значение признака к математическому ожиданию,
тем чаще оно встречается.
Закон распределения вероятностей непрерывной случайной величины
Х называется нормальным, если ее дифференциальная функция f(x)
определяется формулой: ( )
( )
,
2
1 2
2
2σ
πσ
ax
exf
−
−
= где а совпадает с
математическим ожиданием величины Х: а=М(Х), параметр σ совпадает со
средним квадратическим отклонением величины Х: σ = σ(Х). Диаграмма
нормального распределения симметрична относительно точки а
(математического ожидания). Медиана, среднее арифметическое
нормального распределения равны тоже а. При этом в точке а функция f(x)
достигает своего максимума, который равен
πσ 2
1 .
Пример 3: График плотности вероятности нормального
распределения непрерывной величины X изображен на рисунке. Определите
математическое ожидание, среднее квадратическое отклонение и
максимальное значение дифференциальной функции распределения.
Решение.
По графику можно найти
максимальное значение
дифференциальной функции
распределения, оно составляет 0,2.
Функция достигает максимума в точке
x=5, следовательно, математическое
ожидание M(X)=5. В точке максимума
функция плотности вероятности примет
вид: ( )
πσ 2
1
=xf , следовательно,
2
5,22,0
1
14,322,0
1
2)(
1
≈
⋅
=
⋅
==
π
σ
xf
Copyright ОАО «ЦКБ «БИБКОМ» & ООО «Aгентство Kнига-Cервис»
35
В MS Excel для вычисления значений нормального распределения
используются функция НОРМРАСП, которая вычисляет значения
вероятности нормальной функции распределения для указанного среднего и
стандартного отклонения.
Функция имеет параметры:
НОРМРАСП (х; среднее; стандартное_откл; интегральная),
х – значения выборки, для которых строится распределение;
среднее – среднее арифметическое выборки;
стандартное_откл – стандартное отклонение распределения;
интегральная – логическое значение, определяющее форму функции.
Если интегральная имеет значение ИСТИНА(1), то функция НОРМРАСП
возвращает интегральную функцию распределения; если это аргумент имеет
значение ЛОЖЬ (0), то вычисляется значение функции плотности
распределения.
Если среднее = 0 и стандартное_откл = 1, то функция НОРМРАСП
возвращает стандартное нормальное распределение.
Пример 4: Составить дифференциальную функцию распределения
непрерывной величины X, если известно, что величина распределена по
нормальному закону с параметрами: математическое ожидание равно -2, а
среднее квадратическое отклонение 2. Изобразить полученную функцию с
помощью MS Excel.
Решение.
Дифференциальная функция
распределения непрерывной величины X,
распределенной по нормальному закону, имеет
вид: ( )
( )
,
2
1 2
2
2σ
πσ
ax
exf
−
−
= где а –
математическое ожидание; σ – среднее
квадратическое отклонение. По условию задачи математическое
ожидание а=-2; среднее квадратическое отклонение σ=2, следовательно
( )
( )
8
2 2
22
1
+
−
=
x
exf
π
.
Для построения графика необходимо выбрать начальное значение для
переменной x. Серединное значение совпадает с математическим ожиданием
а, начальное значение отстоит от серединного не менее чем на σ, поэтому
примем начальное значение x=-5. Запишем в ячейку A1 значение -5, в ячейку
А2 – формулу =А1+0,2 и «протянем» эту формулу до ячейки А31, в которой
получится значение 1. В ячейку B1 внесем формулу:
=1/(2*КОРЕНЬ(2*ПИ()))*EXP(-((A1+2)^2)/8) и «протянем» эту формулу до
0
0,05
0,1
0,15
0,2
0,25
-8 -6 -4 -2 0 2 4
Copyright ОАО «ЦКБ «БИБКОМ» & ООО «Aгентство Kнига-Cервис»
555.математика и информатика в 2 частях часть 2 практикум
555.математика и информатика в 2 частях часть 2 практикум
555.математика и информатика в 2 частях часть 2 практикум
555.математика и информатика в 2 частях часть 2 практикум
555.математика и информатика в 2 частях часть 2 практикум
555.математика и информатика в 2 частях часть 2 практикум
555.математика и информатика в 2 частях часть 2 практикум
555.математика и информатика в 2 частях часть 2 практикум
555.математика и информатика в 2 частях часть 2 практикум
555.математика и информатика в 2 частях часть 2 практикум
555.математика и информатика в 2 частях часть 2 практикум
555.математика и информатика в 2 частях часть 2 практикум
555.математика и информатика в 2 частях часть 2 практикум
555.математика и информатика в 2 частях часть 2 практикум
555.математика и информатика в 2 частях часть 2 практикум
555.математика и информатика в 2 частях часть 2 практикум
555.математика и информатика в 2 частях часть 2 практикум
555.математика и информатика в 2 частях часть 2 практикум
555.математика и информатика в 2 частях часть 2 практикум
555.математика и информатика в 2 частях часть 2 практикум
555.математика и информатика в 2 частях часть 2 практикум
555.математика и информатика в 2 частях часть 2 практикум
555.математика и информатика в 2 частях часть 2 практикум
555.математика и информатика в 2 частях часть 2 практикум
555.математика и информатика в 2 частях часть 2 практикум
555.математика и информатика в 2 частях часть 2 практикум
555.математика и информатика в 2 частях часть 2 практикум
555.математика и информатика в 2 частях часть 2 практикум
555.математика и информатика в 2 частях часть 2 практикум
555.математика и информатика в 2 частях часть 2 практикум
555.математика и информатика в 2 частях часть 2 практикум
555.математика и информатика в 2 частях часть 2 практикум
555.математика и информатика в 2 частях часть 2 практикум
555.математика и информатика в 2 частях часть 2 практикум
555.математика и информатика в 2 частях часть 2 практикум
555.математика и информатика в 2 частях часть 2 практикум
555.математика и информатика в 2 частях часть 2 практикум
555.математика и информатика в 2 частях часть 2 практикум
555.математика и информатика в 2 частях часть 2 практикум
555.математика и информатика в 2 частях часть 2 практикум
555.математика и информатика в 2 частях часть 2 практикум
555.математика и информатика в 2 частях часть 2 практикум
555.математика и информатика в 2 частях часть 2 практикум
555.математика и информатика в 2 частях часть 2 практикум
555.математика и информатика в 2 частях часть 2 практикум
555.математика и информатика в 2 частях часть 2 практикум
555.математика и информатика в 2 частях часть 2 практикум
555.математика и информатика в 2 частях часть 2 практикум
555.математика и информатика в 2 частях часть 2 практикум
555.математика и информатика в 2 частях часть 2 практикум
555.математика и информатика в 2 частях часть 2 практикум
555.математика и информатика в 2 частях часть 2 практикум
555.математика и информатика в 2 частях часть 2 практикум
555.математика и информатика в 2 частях часть 2 практикум
555.математика и информатика в 2 частях часть 2 практикум
555.математика и информатика в 2 частях часть 2 практикум
555.математика и информатика в 2 частях часть 2 практикум
555.математика и информатика в 2 частях часть 2 практикум
555.математика и информатика в 2 частях часть 2 практикум
555.математика и информатика в 2 частях часть 2 практикум
555.математика и информатика в 2 частях часть 2 практикум
555.математика и информатика в 2 частях часть 2 практикум
555.математика и информатика в 2 частях часть 2 практикум
555.математика и информатика в 2 частях часть 2 практикум
555.математика и информатика в 2 частях часть 2 практикум
555.математика и информатика в 2 частях часть 2 практикум
555.математика и информатика в 2 частях часть 2 практикум
555.математика и информатика в 2 частях часть 2 практикум
555.математика и информатика в 2 частях часть 2 практикум
555.математика и информатика в 2 частях часть 2 практикум
555.математика и информатика в 2 частях часть 2 практикум
555.математика и информатика в 2 частях часть 2 практикум
555.математика и информатика в 2 частях часть 2 практикум
555.математика и информатика в 2 частях часть 2 практикум
555.математика и информатика в 2 частях часть 2 практикум
555.математика и информатика в 2 частях часть 2 практикум
555.математика и информатика в 2 частях часть 2 практикум

More Related Content

What's hot

системы счисления
системы счислениясистемы счисления
системы счисления
kozarezov_vi
 
лекция 4 системы счисления информатика
лекция 4 системы счисления информатикалекция 4 системы счисления информатика
лекция 4 системы счисления информатика
Gulnaz Shakirova
 
Системы счисления
Системы счисленияСистемы счисления
Системы счисления
Tofik Jabiyev
 
представление чисел в компьютере
представление чисел в компьютерепредставление чисел в компьютере
представление чисел в компьютере
Елена Ключева
 
системы счисления
системы счислениясистемы счисления
системы счисления
serg32
 
системы счисления
системы счислениясистемы счисления
системы счисления
kozarezov94
 
Кодирование информации
Кодирование информацииКодирование информации
Кодирование информации
ryabuha
 
лекция 8 арифметические операции информатика
лекция 8 арифметические операции информатикалекция 8 арифметические операции информатика
лекция 8 арифметические операции информатика
Gulnaz Shakirova
 
лекция 8 арифметические операции информатика
лекция 8 арифметические операции информатикалекция 8 арифметические операции информатика
лекция 8 арифметические операции информатика
Gulnaz Shakirova
 
системы счиление
системы счилениесистемы счиление
системы счиление
YUYFF
 
Системы счисления
Системы счисленияСистемы счисления
Системы счисления
Evgeny Smirnov
 

What's hot (17)

системы счисления
системы счислениясистемы счисления
системы счисления
 
лекция 4 системы счисления информатика
лекция 4 системы счисления информатикалекция 4 системы счисления информатика
лекция 4 системы счисления информатика
 
Системы счисления
Системы счисленияСистемы счисления
Системы счисления
 
разбор с
разбор сразбор с
разбор с
 
представление чисел в памяти компьютера
представление чисел в памяти компьютерапредставление чисел в памяти компьютера
представление чисел в памяти компьютера
 
представление чисел в компьютере
представление чисел в компьютерепредставление чисел в компьютере
представление чисел в компьютере
 
системы счисления
системы счислениясистемы счисления
системы счисления
 
системы счисления
системы счислениясистемы счисления
системы счисления
 
Системы счисления
Системы счисленияСистемы счисления
Системы счисления
 
Кодирование информации
Кодирование информацииКодирование информации
Кодирование информации
 
лекция 8 арифметические операции информатика
лекция 8 арифметические операции информатикалекция 8 арифметические операции информатика
лекция 8 арифметические операции информатика
 
лекция 8 арифметические операции информатика
лекция 8 арифметические операции информатикалекция 8 арифметические операции информатика
лекция 8 арифметические операции информатика
 
системы счиление
системы счилениесистемы счиление
системы счиление
 
системы счисления
системы счислениясистемы счисления
системы счисления
 
Системы счисления
Системы счисленияСистемы счисления
Системы счисления
 
Квадратичная математика
Квадратичная математикаКвадратичная математика
Квадратичная математика
 
Математическая индукция
Математическая индукцияМатематическая индукция
Математическая индукция
 

Similar to 555.математика и информатика в 2 частях часть 2 практикум

05
0505
05
JIuc
 
представление числовой информации
представление числовой информациипредставление числовой информации
представление числовой информации
volokobino93
 
9 1.1 - системы счисления
9 1.1 - системы счисления9 1.1 - системы счисления
9 1.1 - системы счисления
jula-mam
 
системы счисления
системы счислениясистемы счисления
системы счисления
AnnaErmakova2013
 
системы счисления
системы счислениясистемы счисления
системы счисления
AnnaErmakova2013
 
системы счисления(CC)
системы счисления(CC)системы счисления(CC)
системы счисления(CC)
AnnaErmakova2013
 
системы счисления
системы счислениясистемы счисления
системы счисления
AnnaErmakova2013
 
системы счисления
системы счислениясистемы счисления
системы счисления
AnnaErmakova2013
 
лекция 8 арифметические операции информатика
лекция 8 арифметические операции информатикалекция 8 арифметические операции информатика
лекция 8 арифметические операции информатика
Gulnaz Shakirova
 

Similar to 555.математика и информатика в 2 частях часть 2 практикум (20)

Кодирование информации
Кодирование информацииКодирование информации
Кодирование информации
 
6 клас 3 урок 2
6 клас 3 урок 26 клас 3 урок 2
6 клас 3 урок 2
 
05
0505
05
 
10
1010
10
 
Sistemy schisleniya munchesku
Sistemy schisleniya muncheskuSistemy schisleniya munchesku
Sistemy schisleniya munchesku
 
Test
TestTest
Test
 
лекция 3.docx
лекция 3.docxлекция 3.docx
лекция 3.docx
 
представление числовой информации
представление числовой информациипредставление числовой информации
представление числовой информации
 
9 1.1 - системы счисления
9 1.1 - системы счисления9 1.1 - системы счисления
9 1.1 - системы счисления
 
Системы счисления
Системы счисленияСистемы счисления
Системы счисления
 
системы счисления
системы счислениясистемы счисления
системы счисления
 
системы счисления
системы счислениясистемы счисления
системы счисления
 
системы счисления(CC)
системы счисления(CC)системы счисления(CC)
системы счисления(CC)
 
двоичная система счисления
 двоичная система счисления двоичная система счисления
двоичная система счисления
 
системы счисления
системы счислениясистемы счисления
системы счисления
 
системы счисления
системы счислениясистемы счисления
системы счисления
 
S1 arithmetics amp-amp_logic
S1 arithmetics amp-amp_logicS1 arithmetics amp-amp_logic
S1 arithmetics amp-amp_logic
 
лекция 8 арифметические операции информатика
лекция 8 арифметические операции информатикалекция 8 арифметические операции информатика
лекция 8 арифметические операции информатика
 
История чисел и систем счисления
   История чисел и систем счисления   История чисел и систем счисления
История чисел и систем счисления
 
Системы счисления
Системы счисленияСистемы счисления
Системы счисления
 

More from efwd2ws2qws2qsdw

720.экология образование туризм подготовка кадров
720.экология образование туризм подготовка кадров720.экология образование туризм подготовка кадров
720.экология образование туризм подготовка кадров
efwd2ws2qws2qsdw
 
719.буддийская и светская этика формирование мировоззрения
719.буддийская и светская этика формирование мировоззрения719.буддийская и светская этика формирование мировоззрения
719.буддийская и светская этика формирование мировоззрения
efwd2ws2qws2qsdw
 
718.детская сибириада «спорт — искусство – интеллект»
718.детская сибириада «спорт — искусство – интеллект»718.детская сибириада «спорт — искусство – интеллект»
718.детская сибириада «спорт — искусство – интеллект»
efwd2ws2qws2qsdw
 
717.история философии хрестоматия
717.история философии  хрестоматия717.история философии  хрестоматия
717.история философии хрестоматия
efwd2ws2qws2qsdw
 
716.psychology in basketball officiating handbook for basketball referees
716.psychology in basketball officiating handbook for basketball referees716.psychology in basketball officiating handbook for basketball referees
716.psychology in basketball officiating handbook for basketball referees
efwd2ws2qws2qsdw
 
715.сборник качественных задач общая педагогика
715.сборник качественных задач общая педагогика715.сборник качественных задач общая педагогика
715.сборник качественных задач общая педагогика
efwd2ws2qws2qsdw
 
714.северная провинция трансформация социальных институтов монография
714.северная провинция трансформация социальных  институтов монография714.северная провинция трансформация социальных  институтов монография
714.северная провинция трансформация социальных институтов монография
efwd2ws2qws2qsdw
 
713.концертные пьесы для русского народного оркестра [ноты] вып 3 партитура
713.концертные пьесы для русского народного оркестра [ноты] вып 3 партитура713.концертные пьесы для русского народного оркестра [ноты] вып 3 партитура
713.концертные пьесы для русского народного оркестра [ноты] вып 3 партитура
efwd2ws2qws2qsdw
 
712.психология эмоционального интеллекта теория, диагностика, практика
712.психология эмоционального интеллекта теория, диагностика, практика712.психология эмоционального интеллекта теория, диагностика, практика
712.психология эмоционального интеллекта теория, диагностика, практика
efwd2ws2qws2qsdw
 
711.дистанционное обучение в высшей школе социально экономический и организац...
711.дистанционное обучение в высшей школе социально экономический и организац...711.дистанционное обучение в высшей школе социально экономический и организац...
711.дистанционное обучение в высшей школе социально экономический и организац...
efwd2ws2qws2qsdw
 
710.seducing the masses an introduction to advertising world
710.seducing the masses an introduction to advertising world710.seducing the masses an introduction to advertising world
710.seducing the masses an introduction to advertising world
efwd2ws2qws2qsdw
 
709.моделирование и анализ транспортных протоколов в информационных сетях мон...
709.моделирование и анализ транспортных протоколов в информационных сетях мон...709.моделирование и анализ транспортных протоколов в информационных сетях мон...
709.моделирование и анализ транспортных протоколов в информационных сетях мон...
efwd2ws2qws2qsdw
 
708.методическое пособие по дисциплине «информатика» ч3 работа с microsoft of...
708.методическое пособие по дисциплине «информатика» ч3 работа с microsoft of...708.методическое пособие по дисциплине «информатика» ч3 работа с microsoft of...
708.методическое пособие по дисциплине «информатика» ч3 работа с microsoft of...
efwd2ws2qws2qsdw
 
707.избранные вопросы обучения геометрии (дистанционные курсы) [текст] учебно...
707.избранные вопросы обучения геометрии (дистанционные курсы) [текст] учебно...707.избранные вопросы обучения геометрии (дистанционные курсы) [текст] учебно...
707.избранные вопросы обучения геометрии (дистанционные курсы) [текст] учебно...
efwd2ws2qws2qsdw
 
706.моделирование нагрузочно измерительных устройств с полыми немагнитными ро...
706.моделирование нагрузочно измерительных устройств с полыми немагнитными ро...706.моделирование нагрузочно измерительных устройств с полыми немагнитными ро...
706.моделирование нагрузочно измерительных устройств с полыми немагнитными ро...
efwd2ws2qws2qsdw
 
705.под часами альманах кн2
705.под часами  альманах  кн2705.под часами  альманах  кн2
705.под часами альманах кн2
efwd2ws2qws2qsdw
 
704.методические основы подготовки судей по баскетболу учебно методическое п...
704.методические основы подготовки судей по баскетболу  учебно методическое п...704.методические основы подготовки судей по баскетболу  учебно методическое п...
704.методические основы подготовки судей по баскетболу учебно методическое п...
efwd2ws2qws2qsdw
 
703.правоведение учебник гриф рао
703.правоведение учебник гриф рао703.правоведение учебник гриф рао
703.правоведение учебник гриф рао
efwd2ws2qws2qsdw
 
702.взаимное страхование в российской федерации экономико организационные асп...
702.взаимное страхование в российской федерации экономико организационные асп...702.взаимное страхование в российской федерации экономико организационные асп...
702.взаимное страхование в российской федерации экономико организационные асп...
efwd2ws2qws2qsdw
 
701.историческое краеведение накопление и развитие краеведческих знаний в рос...
701.историческое краеведение накопление и развитие краеведческих знаний в рос...701.историческое краеведение накопление и развитие краеведческих знаний в рос...
701.историческое краеведение накопление и развитие краеведческих знаний в рос...
efwd2ws2qws2qsdw
 

More from efwd2ws2qws2qsdw (20)

720.экология образование туризм подготовка кадров
720.экология образование туризм подготовка кадров720.экология образование туризм подготовка кадров
720.экология образование туризм подготовка кадров
 
719.буддийская и светская этика формирование мировоззрения
719.буддийская и светская этика формирование мировоззрения719.буддийская и светская этика формирование мировоззрения
719.буддийская и светская этика формирование мировоззрения
 
718.детская сибириада «спорт — искусство – интеллект»
718.детская сибириада «спорт — искусство – интеллект»718.детская сибириада «спорт — искусство – интеллект»
718.детская сибириада «спорт — искусство – интеллект»
 
717.история философии хрестоматия
717.история философии  хрестоматия717.история философии  хрестоматия
717.история философии хрестоматия
 
716.psychology in basketball officiating handbook for basketball referees
716.psychology in basketball officiating handbook for basketball referees716.psychology in basketball officiating handbook for basketball referees
716.psychology in basketball officiating handbook for basketball referees
 
715.сборник качественных задач общая педагогика
715.сборник качественных задач общая педагогика715.сборник качественных задач общая педагогика
715.сборник качественных задач общая педагогика
 
714.северная провинция трансформация социальных институтов монография
714.северная провинция трансформация социальных  институтов монография714.северная провинция трансформация социальных  институтов монография
714.северная провинция трансформация социальных институтов монография
 
713.концертные пьесы для русского народного оркестра [ноты] вып 3 партитура
713.концертные пьесы для русского народного оркестра [ноты] вып 3 партитура713.концертные пьесы для русского народного оркестра [ноты] вып 3 партитура
713.концертные пьесы для русского народного оркестра [ноты] вып 3 партитура
 
712.психология эмоционального интеллекта теория, диагностика, практика
712.психология эмоционального интеллекта теория, диагностика, практика712.психология эмоционального интеллекта теория, диагностика, практика
712.психология эмоционального интеллекта теория, диагностика, практика
 
711.дистанционное обучение в высшей школе социально экономический и организац...
711.дистанционное обучение в высшей школе социально экономический и организац...711.дистанционное обучение в высшей школе социально экономический и организац...
711.дистанционное обучение в высшей школе социально экономический и организац...
 
710.seducing the masses an introduction to advertising world
710.seducing the masses an introduction to advertising world710.seducing the masses an introduction to advertising world
710.seducing the masses an introduction to advertising world
 
709.моделирование и анализ транспортных протоколов в информационных сетях мон...
709.моделирование и анализ транспортных протоколов в информационных сетях мон...709.моделирование и анализ транспортных протоколов в информационных сетях мон...
709.моделирование и анализ транспортных протоколов в информационных сетях мон...
 
708.методическое пособие по дисциплине «информатика» ч3 работа с microsoft of...
708.методическое пособие по дисциплине «информатика» ч3 работа с microsoft of...708.методическое пособие по дисциплине «информатика» ч3 работа с microsoft of...
708.методическое пособие по дисциплине «информатика» ч3 работа с microsoft of...
 
707.избранные вопросы обучения геометрии (дистанционные курсы) [текст] учебно...
707.избранные вопросы обучения геометрии (дистанционные курсы) [текст] учебно...707.избранные вопросы обучения геометрии (дистанционные курсы) [текст] учебно...
707.избранные вопросы обучения геометрии (дистанционные курсы) [текст] учебно...
 
706.моделирование нагрузочно измерительных устройств с полыми немагнитными ро...
706.моделирование нагрузочно измерительных устройств с полыми немагнитными ро...706.моделирование нагрузочно измерительных устройств с полыми немагнитными ро...
706.моделирование нагрузочно измерительных устройств с полыми немагнитными ро...
 
705.под часами альманах кн2
705.под часами  альманах  кн2705.под часами  альманах  кн2
705.под часами альманах кн2
 
704.методические основы подготовки судей по баскетболу учебно методическое п...
704.методические основы подготовки судей по баскетболу  учебно методическое п...704.методические основы подготовки судей по баскетболу  учебно методическое п...
704.методические основы подготовки судей по баскетболу учебно методическое п...
 
703.правоведение учебник гриф рао
703.правоведение учебник гриф рао703.правоведение учебник гриф рао
703.правоведение учебник гриф рао
 
702.взаимное страхование в российской федерации экономико организационные асп...
702.взаимное страхование в российской федерации экономико организационные асп...702.взаимное страхование в российской федерации экономико организационные асп...
702.взаимное страхование в российской федерации экономико организационные асп...
 
701.историческое краеведение накопление и развитие краеведческих знаний в рос...
701.историческое краеведение накопление и развитие краеведческих знаний в рос...701.историческое краеведение накопление и развитие краеведческих знаний в рос...
701.историческое краеведение накопление и развитие краеведческих знаний в рос...
 

555.математика и информатика в 2 частях часть 2 практикум

  • 1. Федеральное агентство по образованию ГОУ ВПО Тульский государственный педагогический университет им. Л. Н. Толстого Р. Р. Яфаева, Ю. И. Богатырева МАТЕМАТИКА И ИНФОРМАТИКА Учебно-методическое пособие Допущено Учебно-методическим объединением по направлениям педагогического образования Министерства образования и науки РФ в качестве учебного пособия для студентов высших учебных заведений, обучающихся по направлению 050700 «Педагогика» В 2 частях Часть 2: ПРАКТИКУМ Тула Издательство ТГПУ им. Л.Н. Толстого 2010 Copyright ОАО «ЦКБ «БИБКОМ» & ООО «Aгентство Kнига-Cервис»
  • 2. 2 ББК 22.1я73+32.81я73 Я89 Рецензенты: доктор физико-математических наук, профессор В. И. Желтков (Тульский государственный университет); кандидат педагогических наук, доцент О. В. Чукаев (Тульский государственный педагогический университет им. Л. Н. Толстого) Яфаева, Р. Р. Я89 Математика и информатика: Учеб.-метод. пособие: В 2 ч. Ч. 2: Практикум / Р. Р. Яфаева, Ю. И. Богатырева.– Тула: Изд-во Тул. гос. пед. ун-та им. Л. Н. Толстого, 2010.– 111 с. В пособии представлены основные положения дисциплины «Математика и информатика», адаптированные для студентов направления подготовки «Педа- гогика». Математика представлена следующими разделами: аксиоматический метод построения математических теорий, комбинаторика, теория множеств, понятия и свойства вероятностей, элементы математической статистики. Информатика представлена разделами: понятие, свойства и измерение инфор- мации; алгоритмы и языки программирования; понятие и компоненты про- граммного и аппаратного обеспечения современной компьютерной техники. Практические задания направлены на формирование умений использовать современные информационные технологии и стандартное программное обеспе- чение в профессиональной деятельности педагога. Представленные примеры решения задач позволяют использовать пособие для организации самостоятель- ной работы студентов. Для студентов высших учебных заведений, обучающихся по направлению подготовки 050700 «Педагогика». ББК 22.1я73+32.81я73 © Р. Р. Яфаева, Ю. И. Богатырева, 2010 © Издательство ТГПУ им. Л. Н. Толстого, 2010 Copyright ОАО «ЦКБ «БИБКОМ» & ООО «Aгентство Kнига-Cервис»
  • 3. 3 II. Практикум Системы счисления Основные умения: осуществлять действия с числами в различных системах счисления. Рекомендуемая литература: [13, 15, 22]. Системы счисления (нумерация) – совокупность способов обозначения натуральных чисел. На ранних ступенях развития общества люди почти не умели считать. Они различали совокупности двух и трех предметов; всякая совокупность, содержавшая бóльшее число предметов, объединялась в понятии «много». Первоначально натуральные числа изображались с помощью некоторого количества черточек или палочек, затем для их изображения стали использовать буквы или специальные знаки. В древнем Новгороде использовалась славянская система, где применялись буквы славянского алфавита; при изображении чисел над ними ставился знак ~ (титло). Древние римляне пользовались нумерацией, сохраняющейся до настоящего времени под именем «римской нумерации», в которой числа изображаются буквами латинского алфавита. Сейчас ею пользуются для обозначения юбилейных дат, нумерации некоторых страниц книги (например, страниц предисловия), глав в книгах, строф в стихотворениях. Выполнение арифметических действий над многозначными числами в этой записи очень трудно. Тем не менее, римская нумерация преобладала в Италии до XIII в., а в других странах Западной Европы – до XVI в. Этим системам свойственны два недостатка, которые привели к их вытеснению другими: необходимость большого числа различных знаков, особенно для изображения больших чисел, и, что еще важнее, неудобство выполнения арифметических операций. Более удобной и общепринятой и наиболее распространенной является десятичная система счисления, которая была изобретена в Индии, заимствована там арабами и затем через некоторое время пришла в Европу. Существовали системы счисления и с другими основаниями. Широкое распространение имела в древности и двенадцатеричная система, происхождение которой, вероятно, связано, как и десятичной системы, со счетом на пальцах: за единицу счета принимались фаланги (отдельные суставы) четырех пальцев одной руки, которые при счете Copyright ОАО «ЦКБ «БИБКОМ» & ООО «Aгентство Kнига-Cервис»
  • 4. 4 перебирались большим пальцем той же руки. Самой молодой системой счисления по праву можно считать двоичную. Различные группы систем счисления см. в части 1: Лекции стр. 12. В непозиционных системах счисления от положения цифры в записи числа не зависит величина, которую она обозначает. Примером непозиционной системы счисления является римская система, в которой в качестве цифр используются латинские буквы. В позиционных системах счисления величина, обозначаемая цифрой в записи числа, зависит от ее позиции. Количество используемых цифр называется основанием системы счисления. Значение числа Х, представленного в виде sp … s1s0, s-1…s-q, равно Х=sp t р +sp-1 t p-1 +…+ s1t + s0 + s-1t -1 +…+s-q t -q , где t – основание системы счисления, равное числу цифр, используемых для записи, si – цифра. Рассмотрим системы счисления: 1) десятичную – t=10, si∈{0,1,…,9}; 2) двоичную – t=2, si∈{0,1}; 3) восьмеричную – t=8, si∈{0,1,…,7}; 4) шестнадцатеричную – t=16, si∈{0,1,…,9,A,B,C,D,E,F}; (416,3)8=4⋅102 +1⋅101 +6⋅100 +3⋅10-1 =400+10+6+0,3= 416,3 (10100)2 = 1⋅24 +0⋅23 +1⋅22 +0⋅21 +0⋅20 =16+4= 20 Перевод в двоичную систему счисления из 10-ичной производится отдельно целой и дробной части. Целая часть – последовательным делением на 2. Остатки от деления, записанные в обратном порядке, образуют новую запись исходного целого числа. Пример 1: Перевести число 92 из десятичной системы в двоичную. 92 : 2 = 46 (ост. 0) 46 : 2 = 23 (ост. 0) 23 : 2 = 11 (ост. 1) 11 : 2 = 5 (ост. 1) 5 : 2 = 2 (ост. 1) 2 : 2 = 1 (ост. 0) 9210=10111002 Дробная часть – последовательным умножением на 2. Цифры в разряде целых образуют искомое представление исходного числа. Copyright ОАО «ЦКБ «БИБКОМ» & ООО «Aгентство Kнига-Cервис»
  • 5. 5 Пример 2: Перевести число 0,648 из десятичной системы в двоичную: 0,648 × 2 = 1,296 (1) 0,296 × 2 = 0,592 (0) 0,592 × 2 = 1,184 (1) 0,184 × 2 = 0,368 (0) 0,368 × 2 = 0,736 (0) 0,736 × 2 = 1,472 (1) и т.д. (0,648)10=(0,101001…)2 Если основание системы счисления k можно представить в виде k=pn , то каждую цифру в записи числа с основанием счисления k заменяется n цифрами системы счисления p. Рассмотрим связь между двоичной, четырехричной, восьмеричной и шестнадцатеричной системами счисления. четырехричная шестнадцатеричная Цифра двоичный код цифра двоичный код 0 00 0 0000 1 01 1 0001 2 10 2 0010 3 11 3 0011 4 0100 5 0101 восьмеричная 6 0110 0 000 7 0111 1 001 8 1000 2 010 9 1001 3 011 A 1010 4 100 B 1011 5 101 C 1100 6 110 D 1101 7 111 E 1110 F 1111 Пример 3: Представим число 92,64810 в 4-й, 8-й и 16-й системах счисления. В двоичной системе число 92,64810 = 1011100,1010012. Число разбивается на группы, группы отчитываются от запятой, разделяющей целую и дробную часть, недостающие позиции заменяются нулями. В начале целой части или в конце дробной нули незначащие. Copyright ОАО «ЦКБ «БИБКОМ» & ООО «Aгентство Kнига-Cервис»
  • 6. 6 Разобьем на пары: 1011100,1010012 = 01 01 11 00,10 10 01 = 1130,2214 Разобьем на триады: 1011100,1010012 = 001 011 100,101 001 = 134,518 Разобьем на тетроды: 1011100,1010012 = 0101 1100,1010 0100 = 5С,A416 92, 64810 = 1011100,1010012= 1130,2214= 134,518 =5С,A416 Можно заметить, что чем больше основание системы счисления, тем меньше необходимо цифр для записи числа. Арифметические операции Арифметические операции во всех позиционных системах счисления выполняются по правилам, аналогичным в десятичной системе. Для выполнения операций необходимы следующие понятия: A M B=C (mod D) 1) остаток от деления (D); 2) целочисленное деление (C). Например, 8 M 5 =1 (mod 3); 10 M 3 = 3 (mod 1); 8 M 4 = 2 (mod 0). Сложение При сложении, числа записываются столбиком в соответствии с разрядами. Складываются цифры. Записывается цифра, равная остатку от деления суммы на основание системы счисления, а число, равное результату целочисленного деления суммы на основание системы счисления переносится в старший разряд. При сложении чисел можно воспользоваться таблицей: + 0 1 0 0 1 1 1 10 Пример 4: Сложить числа 110110102 и 1010102. Решение. Проверка: 110110102 =218; 1010102= 42; 1000001002=260; 218+42=260 Выполнить действия: 1000112+101112=1110102; 101112 + 1012=111002; 110110102 + 1010102=1000001002; 101,012 + 11,112=10012; 4758+5148=12118; 728 + 128=1048; 16,248 + 53,458=71,718 . 1 1 1 1 1 1 1 0 1 1 0 1 02 + 1 0 1 0 1 02 1 0 0 0 0 0 1 0 02 Copyright ОАО «ЦКБ «БИБКОМ» & ООО «Aгентство Kнига-Cервис»
  • 7. 7 Вычитание Для осуществления операции вычитания можно воспользоваться таблицей сложения. Возможен заем числа равного основанию системы счисления из старших разрядов, при этом нули, стоящие между цифрой, от которой вычитают и цифрой, у которой «занимают» превращаются в максимальную цифру системы счисления. Пример 5: Вычесть из числа 11100112 число 11102. Решение. . 1 2 1 1 1 0 0 1 12 – 1 1 1 02 1 1 0 0 1 0 12 Проверка: 11100112= 115; 11102 = 14; 11001012=101; 115-14=101 Выполнить действия: 10101112 -11012 =10010102; 5148 – 4758=178; 728 – 168=548; 53,158 – 16,268=34,678 Умножение При умножении числа записываются столбиком, перемножается каждая цифра множителя на каждую цифру множимого числа, записывается остаток от деления результата умножения на основание системы счисления, а целая часть складывается с результатом умножения следующей цифры, затем полученный столбец чисел складывается. Для умножения чисел можно воспользоваться таблицей: Двоичная система: × 0 1 0 0 0 1 0 1 Пример 6: Найти произведение чисел 110112 и 1102. Решение. 1 1 0 1 12 х 1 1 02 1 1 0 1 1 1 1 0 1 1 1 0 1 0 0 0 1 02 Проверка: 110112 =27 ; 1102=6 ; 101000102 = 162; 27×6=162. Выполнить действия: (11011)2 × (110)2= (10100010)2 (33) 8 × (6)8= (242)8 (1В) 16 × (6)16= (А2)16 Copyright ОАО «ЦКБ «БИБКОМ» & ООО «Aгентство Kнига-Cервис»
  • 8. 8 Деление Деление осуществляется по тому же алгоритму, что и в десятичной системе – «деление уголком», также можно воспользоваться таблицей умножения. От делимого выделяется часть большая делителя, но не больше чем в t раз (t – основание системы счисления). В результате подбирается цифра, произведение делителя на которую даст число, меньшее выделенного, произведение записывается под делимым, сносится следующая цифра, если получившееся значение числа превосходит делитель, пишется – подбирается новая цифра, если нет – в частном пишется 0 и сносится следующая цифра и т.д., до получения результата или достижения требуемой точности. При проведении арифметических операций над числами, выраженными в различных системах счисления, необходимо предварительно перевести их в одну и ту же систему. Пример 7: Найти частное чисел 10000012 и 1012. Решение. Проверка: 10000012 =65; 1012= 5; 11012 = 13; 65: 5= 13. Выполнить действия: (10100010)2 : (110)2= (11011)2 (242)8 : (6)8= (33)8 (А2)16 : (6)16=(1В) 16 Задания для самостоятельного выполнения Задание 1. Перевести в десятичную систему: 1001012 =____________ 10 131,58 =______________10 A0C416 =_____________10 2031,024 =____________10 Задание 2. Перевести в различные системы счисления из десятичной: 93310 =_________________16 45,8310 =________________8 68810 =_____________________8 27,7210 =____________________2 1 0 0 0’ 0 0 12 1 0 12 1 0 1 1 1 0 12 1 1 0 1 0 1 1 0 1 1 0 1 0 Copyright ОАО «ЦКБ «БИБКОМ» & ООО «Aгентство Kнига-Cервис»
  • 9. 9 29510 =_____________________16 83,0310 =____________________4 Задание 3. Выполнить действия, проверить путем перевода в десятичную систему счисления: 100112-1102=_____________________2 100112+101112 =__________________2 111012×1012 =____________________2 1110002/11102 =___________________2 10001112-101102 =_________________2 100112×101112 =___________________2 10000112+10012 =__________________2 110012/1012 =______________________2 Задание 4. Записать в различных системах счисления, используя таблицы: 1001001,1102 =_________________4 10011,101112 =_________________16 11101,1012 =___________________8 543,138 =___________________2 15A,C316 =________________________2 416,1138 =___________________2 232,0014 =__________________2 7C1,5916 =______________________2 Вопросы для самоконтроля 1. В чем отличие позиционной системы счисления от непозиционной? 2. Приведите примеры непозиционных систем счисления. 3. Составьте алгоритм выполнения сложения в двоичной системе счисления. 4. Составьте алгоритм выполнения вычитания в двоичной системе счисления. 5. Составьте алгоритм выполнения умножения в двоичной системе счисления. 6. Составьте алгоритм выполнения деления в двоичной системе счисления. Copyright ОАО «ЦКБ «БИБКОМ» & ООО «Aгентство Kнига-Cервис»
  • 10. 10 Индивидуальные задания 1. Перевести в десятичную систему: 1001012 1011,112 0,11012 Перевести числа в двоичную, восьмеричную и шестнадцатеричную систему счисления из десятичной с точностью до 8 знаков после запятой для бесконечных дробей: 3310 45,8310 0,12510 Выполнить действия, проверить путем перевода в десятичную: 101110012-1000112 ; 100112+101112; 100112×10112 ; 100001110002/11112. 2. Перевести в десятичную систему: 1101112 1001,012 0,1012 Перевести числа в двоичную, восьмеричную и шестнадцатеричную систему счисления из десятичной с точностью до 8 знаков после запятой для бесконечных дробей: 6810 0,7510 27,7210 Выполнить действия, проверить путем перевода в десятичную: 10001010112-111012 ; 100112×101112; 10110112+101111012 ; 101001000002/100002 3. Перевести в десятичную систему: 101101102 1011,1012 0,111012 Перевести числа в двоичную, восьмеричную и шестнадцатеричную систему счисления из десятичной с точностью до 8 знаков после запятой для бесконечных дробей: 62910 0,2510 19,3910 Выполнить действия, проверить путем перевода в десятичную: 1011000012-10102 ; 111012+101012; 1011101102×1000102 ; 100001110002/111100. 4. Перевести в десятичную систему: 1101002 101001,12 0,11012 Перевести числа в двоичную, восьмеричную и шестнадцатеричную систему счисления из десятичной с точностью 8 знаков после запятой для бесконечных дробей: 29510 0,62510 83,0310 Выполнить действия, проверить путем перевода в десятичную: 10011000012-1011112; 1001012+101112; 101102×10112; 1010002/10002 Copyright ОАО «ЦКБ «БИБКОМ» & ООО «Aгентство Kнига-Cервис»
  • 11. 11 Основные математические структуры Основные умения: применять основные математические структуры и арифметические операции над ними к решению профессиональных задач. Рекомендуемая литература: [7, 12, 15, 18]. Множества Понятие множества, подмножества, свойства и операции над множествами см. в части 1 «Лекции» стр. 18-21. Пример 1: В классе 12 учеников, которые хорошо играют в волейбол, и 9 учеников, которые хорошо играют в теннис, всего в классе 16 человек. Сколько учеников хорошо играют и в волейбол, и в теннис? Решение. Количество волейболистов V=12; количество теннисистов Т=9; количество волейболистов и теннисистов 16=∪ TV . Найти количество учеников, хорошо играющих и в волейбол и в теннис ?−∩ TV .57416 ;7916)( ;41216)(| );()()( =−−≡∩ =−≡∪= =−≡∪= ∪=∩ TV TTVTV VTVVT VTTVTVTV Пример 2: Из 100 студентов экзамен по педагогике сдали 28 человек, математике – 30 человек, философии – 42 человека, педагогику и математику сдали 8 человек, математику и философию – 5, педагогику и философию – 10, все три экзамена – 3 человека. Сколько студентов не сдали ни одного экзамена? Решение. Количество студентов, сдавших педагогику Р=28; количество студентов, сдавших математику М=30; количество человек, сдавших философию F=42; количество человек, сдавших педагогику и математику 8;P M∩ = количество человек, сдавших математику и философию ;5=∩ FM количество 12 9 ? 28 308 42 10 5 3 Copyright ОАО «ЦКБ «БИБКОМ» & ООО «Aгентство Kнига-Cервис»
  • 12. 12 человек, сдавших педагогику и философию ;10=∩ FP количество человек, сдавших педагогику, математику и философию ;3=∩∩ FMP всего человек U=100. Найти количество человек, не сдавших ни одного экзамена: ))())(())(())(( )()()(( FMPMFPPFMFMP PMFFPMFMPU ∩∩∪∩∪∩∪∩ ∪∪∪ . Студенты, сдавшие 3 экзамена: 3.P M F∩ ∩ = Студенты, сдавшие 2 экзамена: педагогику и математику: ;538)( =−≡∩ FMP математику и философию: 235)( =−≡∩ PFM ; педагогику и философию: .7310)( =−≡∩ MFP Студенты, сдавшие 1 экзамен: педагогику – PMF ≡ 28-5-7-3=13; математику: MPF ≡ 30-5-2-3=20; философию: FMP ≡ 42-2-7-3=30. Студенты, не сдавшие ни одного экзамена: 100-(13+20+30+5+2+7+3)=100-80=20. Пример 3: 35 учеников лингвистической гимназии поехали в тур по Европе. Каждый из гимназистов владеет хотя бы одним иностранным языком, 25 учеников знают английский язык, 15 – немецкий, 20 – французский, 15 гимназистов знают английский и французский, 6 – немецкий и французский, 10 – немецкий и английский языки. Сколько учеников знают: а) все языки б) только немецкий и французский, но не знают английского? Решение: Языки Английский Французский Немецкий Английский X U V Французский Y T Немецкий Z X – знают только английский; Y – знают только французский; Z – знают только немецкий; U – знают только английский и французский; T – знают только французский и немецкий; V – знают только английский и немецкий; W – множество учеников, знающих все 3 языка; Всего учеников – 35 человек: X+Y+Z+U+V+T+W=35 X+U+V+W=25; Y+U+T+W=20; Z+V+T=W=15 U+W=15 => U=15-W T+W=6 => T=6-W V+W=10 => V=10-W X+15-W+10-W+W=25 => X+25-W=25 => X=W Copyright ОАО «ЦКБ «БИБКОМ» & ООО «Aгентство Kнига-Cервис»
  • 13. 13 Y+15-W+6-W+W=20 => Y+21-W=20 => Y=W-1 Z+10-W+6-W+W=15 => Z=W-1 W+W-1+W-1+15-W+6-W+10-W+W=35 => W=6 U=9; T=0; V=4; X=6; Y=5; Z=5 Языки Английский Французский Немецкий Английский 6 9 4 Французский 5 0 Немецкий 5 Ответ: 6 учеников знают все три языка; 10 учеников знают немецкий или французский и не знают английского Задания для самостоятельного выполнения Задание 1. Из группы студентов на занятия физкультурой ходят 20 человек, а в секции – 18 человек, причем 15 человек одновременно ходят и в секции и на занятия по физкультуре. Сколько студентов освобождены от занятий спортом, если всего в группе 25 человек? Задание 2. Группа учащихся из 100 человек собралась в тур по Европе. 70 человек знают английский язык, 45 знают французский язык и 23 человека знают оба языка. Сколько человек в группе не знают ни английского, ни французского языка? Задание 3. Из 40 учеников 30 умеют играть в шашки, 27 умеют играть в шахматы и только пятеро не умеют ни того, ни другого. Сколько учеников умеют играть и в шашки и в шахматы? Задание 4. 20 мальчиков поехали на пикник. При этом 5 из них обгорели, 8 были сильно покусаны комарами, а 10 остались всем довольны. Сколько обгоревших мальчиков не было покусано комарами? Сколько покусанных комарами мальчиков также и обгорели? Задание 5. В библиотеке вуза читатели обычно берут либо один учебник по педагогике, либо один задачник по математике, либо один учебник и один задачник. В один из дней было выдано 57 учебников и 36 задачников. Сколько было читателей в библиотеке, если 12 человек взяли и учебник и задачник? Вопросы для самоконтроля 1. Приведите примеры множеств в предметной области Педагогика? 2. Приведите пример подмножества из множества всех студентов факультета психологии? 3. Назовите свойство, которым связаны элементы множества. 4. Какое множество называется универсальным? 5. Какие операции над множествами существуют? 6. Какое множество называют пустым? 7. Приведите пример бесконечного множества. Copyright ОАО «ЦКБ «БИБКОМ» & ООО «Aгентство Kнига-Cервис»
  • 14. 14 Индивидуальные задания 1. Числовые множества представлены в виде промежутков. Найдите и изобразите полученные промежутки с помощью графического редактора Paint: [8; 15] ∩ [9; 20]; (-1; 1) ∩ [-1; 0); [-1; 1] ∩ [-1; 0]; (-1; 0] ∩ [1; +∞); [-1; 0) ∪ [0; 4]; [1; +∞) ∪ [0; +∞); {4}∪ (-∞; 4); (0; 2) ∪ [0; 2]. 2. На учебе студенческого актива в «Алексин-Боре» фотокорреспондент университетской газеты взял 2 пленки в 36 и 24 кадра. После печати фотографий выяснилось, что фотографий со студентами ФФК получилось – 40, со студентами ФИСиГН – 20, со студентами исторического факультета – 15. В числе которых было 10 фотографий на которых запечатлены студенты ФФК и ФИСиГН, 10 – ФИиП и ФФК, 5 – ФИСиГН и ФИиП и 1 общая фотография. Сколько фотографий получилось, где запечатлены только студенты ФФК, только ФИСиГН и только исторического факультета? Сколько фотографий не получилось? 3. Составьте множество двухзначных чисел, в записи которых используются лишь цифры 2, 5 и 8. Найдите пересечение этого множества с множеством четных чисел. 4. В 11 «А» классе готовились к выпускному вечеру. Всего в классе 32 человека, из них 17 учащихся написали стихи, 19 – рассказ о школе. Сколько учеников сочинили и стих, и рассказ? 5. Из класса в 20 человек каждый ходит на какую-нибудь секцию, 15 человек – в спортивную, 10 – в танцевальную и 11 – занимаются хоровым пением. Причем 4 человека посещают танцы и хоровое пение, 7 – спортивную и танцевальную секцию, 8 человек посещают хоровое пение и спортивную секцию. Составить таблицу распределения посещения различных секций учениками класса. Copyright ОАО «ЦКБ «БИБКОМ» & ООО «Aгентство Kнига-Cервис»
  • 15. 15 Комбинаторика Основные формулы комбинаторики более подробно разобраны в части «Тезисы лекций» стр. 22-24. Правило суммы Пусть имеется n попарно непересекающихся множеств A1, A2,…,An содержащих m1, m2,…,mn элементов соответственно. Число способов, которыми можно выбрать один элемент из всех этих множеств, равно m1 + m2 +…+ mn Пример 1: На курсе имеется 3 группы. В первой – 25 человек, во второй – 30, в третьей – 20. Сколькими способами из них можно выбрать одного студента? Правило произведения Пусть имеется n множеств A1, A2,…,An, содержащих m1, m2,…,mn элементов соответственно. Число способов, которыми можно выбрать по одному элементу из каждого множества, то есть построить кортеж (а1, а2, …, аn), где аi ∈ Аi, i=1,2,…,n, равно m1ּ m2 ּ …ּmn. Пример 2: На курсе имеется 3 группы. В первой – 25 человек, во второй – 30, в третьей – 20. Сколькими способами из каждой из них можно выбрать по одному студенту? Перестановки без повторений Различные кортежи, которые можно построить из элементов данного множества, взятых ровно по одному разу, называются перестановками. !nPn = Факториал (n!) – это последовательное произведение натуральных чисел от 1 до n: n!=1⋅2⋅…⋅ n. Пример 3: Сколькими способами можно расставить в шеренгу студентов группы из 25 человек? Размещения без повторений Различные упорядоченные подмножества по m элементов данного множества, содержащего n элементов, называются размещениями из n по m. ( )! ! mn n Am n − = Пример 4: Из класса, состоящего из 25 учащихся, необходимо выбрать команду из четырех человек на шахматный турнир на I, II, III и IV доски. Сколькими способами это можно сделать? Copyright ОАО «ЦКБ «БИБКОМ» & ООО «Aгентство Kнига-Cервис»
  • 16. 16 Сочетания без повторений Различные неупорядоченные подмножества по m элементов из данного множества, содержащего n элементов, называются сочетаниями из n по m. ( )!! ! mnm n C m n − = В частности, С 1 n = n; С 0 n = 1, С n n = 1. Пример 5: Сколькими способами из группы в 25 человек можно выбрать команду из пяти человек на олимпиаду по математике? Размещения с повторениями Различные кортежи длины m, составленные из элементов данного множества, содержащего n элементов, так, что эти элементы в кортеже могут повторяться, называются размещениями с повторениями из n по m. mm n nA = ~ Пример 6: Группа студентов педагогического вуза, состоящая из 25 человек сдает экзамен. Возможные оценки – 2, 3, 4, 5. Сколькими способами может быть заполнена экзаменационная ведомость? Перестановки с повторениями Составом кортежа длины n, состоящего из элементов a1,a2,…,ak, называется кортеж (n1, n2,…,nk), где ni – число вхождений элемента аi, i=1, 2,…,k, в данный кортеж, причем n1+n2+…+nk=n. Перестановки с повторениями – это различные кортежи данного состава. !!! ! ),...,,( 21 21 k kn nnn n nnnP K⋅ = Пример 7: Сколько различных «слов» можно составить, переставляя буквы слова «математика»? Сочетания с повторениями Различные неупорядоченные наборы, составленные из m элементов данного множества так, что элементы в наборе могут повторяться, и порядок их не важен, называются сочетаниями с повторениями из n по m. m nС ~ = )!1n(!m )!1mn( − −+ Пример 8: Группа студентов педагогического вуза, состоящая из 25 человек, сдала экзамен. В отчете показано, сколько каких оценок было получено. Сколькими способами может быть составлен отчет? Copyright ОАО «ЦКБ «БИБКОМ» & ООО «Aгентство Kнига-Cервис»
  • 17. 17 Алгоритм решения комбинаторных задач При решении комбинаторных задач следует ответить на следующие вопросы: 1. Из какого множества осуществляется выбор (найти n – количество элементов из которых составляются комбинации)? 2. Сколько элементов в одной комбинации (найти m; если n=m, то применяем правило перестановок P и переходим к вопросу 4)? 3. Важен ли порядок (изменится ли комбинация, если в ней поменять элементы местами)? если важен, применяем правило размещений А, если нет – сочетаний С. 4. Возможны ли повторения элементов в одной комбинации? Рекомендуемая литература: [7, 12, 15, 18]. Пример 9: В фортепианном кружке дома детского творчества занимается 10 человек, в кружке художественного слова – 15, в вокальном – 12 и в фотокружке – 20. Сколькими способами можно составить команду из 4 чтецов, 3 пианистов, 5 певцов и одного фотографа для выезда на экскурсию? Решение. Разобьем решение задачи на подзадачи. 1. Сначала найдем сколькими способами можно выбрать чтецов. 1) производим выбор из 15 человек, n=15; 2) выбираем 4 человек, m=4; 3) порядок не важен, т.е. используем правило сочетаний С 4 15; 4) сочетания без повторений, так как люди выбираются разные. 2. Проводя подобные рассуждения, выбираем пианистов: 3 из 10 – С 3 10 способов. 3. Певцов: 5 из 12 – С 5 12 способов. 4. Фотографа: 1 из 20 – С 1 20 способов. Поскольку выбор производится по всем четырем позициям, а не по одной, применяем правило произведения: 91 20 5 12 3 10 4 15 10595,220 )!512(!5 !12 )!310(!3 !10 )!415(!4 !15 ⋅=⋅ − ⋅ − ⋅ − =⋅⋅⋅ CCCC Ответ: команду можно составить 2,595ּ109 способами. Пример 10: В классе 15 мальчиков и 20 девочек. Для концерта надо выделить танцевальный дуэт, дуэт певцов, дуэт гимнастов. Сколькими способами это можно сделать при условии, что все дети могут танцевать, петь, выполнять гимнастические упражнения, а дуэт должны составлять мальчик и девочка? Copyright ОАО «ЦКБ «БИБКОМ» & ООО «Aгентство Kнига-Cервис»
  • 18. 18 Решение. Разобьем задачу на подзадачи. 1. Найдем, сколькими способами для концерта можно выбрать мальчиков. 1) n=15; 2) m=3; 3) порядок важен, так как в списке фамилии будут записаны по порядку, кто – гимнаст, кто – певец и т.д. То есть применим правило размещений А 3 15 ; 4) повторения невозможны, так как одного человека нельзя выбрать несколько раз. 2. Найдем сколькими способами для концерта можно выбрать девочек. 1) n=20; 2) m=3; 3) порядок важен; 4) повторения невозможны. Для того чтобы найти окончательный ответ, перемножим полученные выражения (применяем правило произведения потому, что выбираются и мальчик, и девочка): А 3 15 ּ А 3 20 =2730 ּ6840=18673200 Ответ: три дуэта можно выбрать 18673200 способами. Задания для самостоятельного выполнения Задание 1. Сколькими способами 5 человек могут построиться в шеренгу? n=____ m=_____ Порядок важен?______ Повторения __________ Формула: __________________ Ответ: ___________________ Задание 2. Сколькими способами из группы детей начального школьного возраста в 18 человек можно выбрать двоих участников детского утренника? n=____ m=_____ Порядок важен?______ Повторения __________ Формула: __________________ Ответ: ___________________ Задание 3. Сколькими способами из группы студентов 18 человек можно выбрать актив группы: старосту, зам. старосты, физорга и затейника. n=____ m=_____ Порядок важен?______ Повторения __________ Формула: __________________ Ответ: ___________________ Copyright ОАО «ЦКБ «БИБКОМ» & ООО «Aгентство Kнига-Cервис»
  • 19. 19 Задание 4. Сколько шестибуквенных слов можно составить из карточек, из которых составлено слово «ананас»? n=____ m=_____ Порядок важен?______ Повторения __________ Формула: __________________ Ответ: ___________________ Задание 5. Сколько существует различных пятизначных телефонных номеров, если первая цифра номера всегда 2? Задание 6. Сколькими способами можно расставить 12 черных и 12 белых шашек на 32 черных клетках шахматной доски? Вопросы для самоконтроля 1. Какие задачи называются комбинаторными? 2. Сформулируйте правило произведения. 3. Сформулируйте правило суммы. 4. Расскажите алгоритм решения комбинаторных задач. Индивидуальные задания 1. В библиотеке на полке стоят 5 книг. Сколько существует различных способов их расположения? 2. Сколькими способами в команде КВН, состоящей из 10 человек, можно выбрать капитана и ведущего? 3. Сколькими способами из группы спортсменов в 12 человек можно выбрать двоих участников соревнования? 4. Сколько различных слов (не обязательно осмысленных) можно получить перестановками всех букв в слове РОТОР, ТОПОР? 5. Для школы учителю необходимо купить тетради в линейку. В магазине продаются 7 различных видов тетрадок. Сколько существует способов покупки 4 тетрадок. 6. В третьем классе 13 мальчиков и 12 девочек. Для концерта необходимо создать три дуэта, в каждом из них должен быть один мальчик и одна девочка. Сколькими способами это можно сделать? 7. 7 девочек водят хоровод. Сколькими различными способами они могут встать в круг? 8. Сколько трехзначных чисел можно составить из цифр 1,2,3,4,0, если цифры в числе могут повторяться? 9. На урок физкультуры в 1 классе пришли 5 учеников. На первом занятии учитель обучает их, как рассчитаться по порядку. По команде «По порядку номеров рассчитайся» учащиеся производят расчет: «первый – второй – третий – четвертый – пятый». После этого учитель перестраивает их Copyright ОАО «ЦКБ «БИБКОМ» & ООО «Aгентство Kнига-Cервис»
  • 20. 20 по-новому и расчет повторяется. Сколько раз может учитель физкультуры повторить это упражнение, используя только разные способы построения первоклассников? 10. Сколькими способами можно расселить 8 студентов в одно-, трех- и четырехместной комнатах? 11. Сколькими способами из группы в 24 человека можно выбрать двоих делегатов на конференцию. 12. В почтовом отделении продаются открытки 10 видов. Сколькими способами можно купить 12 открыток? 13. Телефонные номера в городе состоят из шести цифр. Сколько всего телефонных номеров может быть в городе, если номер начинается только на цифру 5? 14. На почте имеются марки 5 различных типов. Сколько существует способов покупки 10 марок? 15. Сколькими различными способами могут разместиться на скамейке 4 человека? 16. Сколько различных шестизначных чисел можно записать с помощью цифр: 1,1,1,2,2,2? 17. Однажды утром по улице города на высокой скорости пронеслась машина. Она сбила зазевавшегося поросенка и скрылась в неизвестном направлении. Возвращавшийся из ресторана житель города заметил номер автомобиля. Но когда появилась милиция, он с перепугу вспомнил буквы номера и, что все цифры номера нечетные и различные, но точно не помнил их расположения. Сколько номеров нужно проверить по картотеке ГИБДД, чтобы найти нарушителя, если в номере 3 цифры? Copyright ОАО «ЦКБ «БИБКОМ» & ООО «Aгентство Kнига-Cервис»
  • 21. 21 Определения и свойства вероятности Основные умения: решать задачи на определение вероятности; научиться определять вероятность суммы и произведения событий; выделять гипотезы и применять к решению задач формулы полной вероятности и Бейеса. Рекомендуемая литература: [7, 8, 12, 15, 18]. Определения вероятности Наблюдение явления, опыт, эксперимент, которые можно провести многократно, в теории вероятностей принято называть испытанием. Результат, исход испытания, называется событием. Событие называется случайным, если его наступление или ненаступление в некотором испытании (эксперименте) зависит от ряда случайных факторов. Понятие о случайном событии см. в части 1 «Лекции» стр. 25. Совокупность образует полную группу событий для данного испытания, если его результатом обязательно становится хотя бы одно из них. События, образующие полную группу попарно несовместных и равновозможных событий, будем называть элементарными событиями. Классическое определение вероятности Вероятностью P(A) события A называется отношение числа элементарных событий m, благоприятствующих событию A, к числу всех элементарных событий n: n m AP =)( Пример 1: В урне 20 шаров: 15 белых и 5 черных. Какова вероятность извлечь черный шар, если вынимается один шар? Решение. Пусть событие А – вынуть черный шар, тогда m = 5 – количество черных шаров, n = 20 – количество всех шаров. 5 1 ( ) 0,25 20 4 m P A n = = = = Статистическое определение вероятности Пусть проводятся n опытов событие A наступило m раз, тогда n m AP =)( , где m – абсолютная частота события A, P(A) – относительная частота события A. Copyright ОАО «ЦКБ «БИБКОМ» & ООО «Aгентство Kнига-Cервис»
  • 22. 22 Gмера gмера AP =)( Вероятностью события А для испытания в данном опыте называется число Р(А), около которого группируются значения относительной частоты при больших n. Пример 2: Французский естествоиспытатель XVIII в. Ж.Л. Бюффон при экспериментальной проверке закона больших чисел бросил монету 4 040 раз, в результате чего герб выпал 2 048 раз. Найти относительную частоту выпадения герба в данном эксперименте. Решение. Событие А – выпадение герба, абсолютная частота появления герба m = 2048, общее количество n = 4040, тогда ( ) 51,0 4040 2048 ≈==ΑΡ n m Геометрическое определение вероятности Области могут быть различной размерности (одно-, двух- или трехмерного измерения), и в зависимости от выбора размерности меры могут принимать значения либо длины, либо площади, либо объема. Для конкретного испытания размерность мер g и G должна быть одна. Пример 3. В квадрат со стороной 1 см случайным образом бросается точка, положение которой равновозможно в любом месте квадрата. Какова вероятность, что расстояние от нее до центра квадрата не больше 0,5 см? Решение. Область, в которую должна попасть точка, представляет собой круг с центром в точке пересечения диагоналей квадрата и радиусом 0,5 см. Следовательно, S – площадь квадрата, S1 – площадь круга. Событие А – точка попала в круг. S= 12 см2 . S1 = πR2 = 3,14 · (0,5)2 =3,14·0,25=0,785 см2 . Вероятность попадания точки в искомую область равна: 785,0 1 785,0 )( 1 === S S AP Задания для самостоятельного выполнения Задание 1. Из колоды карт в 52 штуки вынимают одну. Какова вероятность извлечь из колоды любую карту крестовой масти? A – _____________________________________________________ n=________ m=_______ Формула: _______________Ответ: _______ Copyright ОАО «ЦКБ «БИБКОМ» & ООО «Aгентство Kнига-Cервис»
  • 23. 23 Задание 2. Бросают игральную кость. Найти вероятность того, что на верхней грани появится четное число очков? A – _____________________________________________________ n=________ m=_______ Формула: _______________Ответ: _______ Задание 3. В урне 4 белых и 3 черных шара. Наугад вынимают два шара. Какова вероятность, что они разного цвета? A – _____________________________________________________ n=________ m=_______ Формула: _______________Ответ: _______ Задание 4. Среди 25 учеников класса, в котором 10 девочек, разыгрывается 5 билетов в цирк. Найти вероятность того, что среди обладателей билетов окажется 2 девочки? A – _____________________________________________________ n=________ m=_______ Формула: _______________Ответ: _______ Вопросы для самоконтроля 1. Что такое частота событий? 2. Что такое вероятность события? 3. Чему равна вероятность достоверного события? 4. Чему равна вероятность невозможного события? 5. В каких пределах заключена вероятность события? 6. Какое определение вероятности называют статистическим? Индивидуальные задания 1. Все натуральные числа от 1 до 30 записаны на одинаковых карточках и помещены в урну. После тщательного перемешивания карточек из урны извлекается одна карточка. Какова вероятность того, что число на взятой карточке окажется кратным 5? 2. В урне лежат 4 белых и 3 черных шара. Наугад вынимаются 2 шара. Какова вероятность, что они черного цвета? 3. В группе 5 юношей и 3 девушки. Среди них по жребию разыгрываются два билета в кино. Какова вероятность, что билеты достанутся юноше и девушке? 4. В круг радиуса 2 случайным образом бросается точка, положение которой равновозможно в любом месте круга. Какова вероятность, что расстояние от нее до центра круга не больше 1,5? Copyright ОАО «ЦКБ «БИБКОМ» & ООО «Aгентство Kнига-Cервис»
  • 24. 24 5. В ящике находятся 15 красных, 9 голубых и 6 зеленых шаров. Из ящика подряд вынимаются шары без возвращения обратно. Какова вероятность того, что из 6 вынутых шаров окажется 1 зеленый, 2 голубых и 3 красных шара? 6. Из карточек, из которых составлено слово «дисплей», случайным образом выбраны три и выложены в ряд. Какова вероятность, что они образовали слово «лес»? 7. Студент сдает в сессию три экзамена. За каждый экзамен равновозможны оценки «5», «4», «3», «2». Стипендия назначается при отсутствии оценок «3» и «2». Какова вероятность назначения стипендии? 8. Участники жеребьевки тянут из ящика жетоны с номерами от 1 до 100. Найти вероятность того, что номер первого наудачу извлеченного жетона не содержит цифры 5. 9. В куб с ребром 1 случайным образом бросается точка, положение которой равновозможно в любом месте куба. Какова вероятность, что расстояние от нее до выделенной вершины куба больше 0,5? 10. На окружности случайным образом выбрали две точки, положение которых равновозможно в любом месте окружности. Какова вероятность, что расстояние между ними больше радиуса окружности? 11. На 5 одинаковых карточках написаны буквы Б, Е, Р, С, Т. Эти карточки наудачу разложили в ряд. Какова вероятность того, что получилось слово БРЕСТ? 12. В квадрат со стороной 2 случайным образом бросается точка, положение которой равновозможно в любом месте квадрата. Какова вероятность, что расстояние от нее до центра квадрата больше 0,5? 13. Среди тысячи новорожденных оказалось 517 мальчиков. Найти относительную частоту рождения мальчиков. 14. Студент из 20 вопросов к зачету усвоил 16. Билет состоит из двух вопросов. Какова вероятность, что он не знает ответа ни на один из доставшихся ему вопросов? 15. В круг радиуса 3 случайным образом бросается точка, положение которой равновозможно в любом месте круга. Какова вероятность, что расстояние от нее до центра круга не больше 1? Copyright ОАО «ЦКБ «БИБКОМ» & ООО «Aгентство Kнига-Cервис»
  • 25. 25 Свойства вероятности, формула полной вероятности, формула Бейеса Произведением событий А и В называется событие С=А⋅В, состоящее в том, что в результате испытания произошло и событие А и событие В, т.е. оба события произошли. Два события A и B называются независимыми, если вероятность появления каждого из них не зависит от того, появилось другое событие или нет. В противном случае события A и B называются зависимыми. Теорема 1. Вероятность произведения двух независимых событий A и B равна произведению этих вероятностей. P(A⋅B)=P(A) ⋅P(B) Пусть А и В – зависимые события. Условной вероятностью PA(B) события В называется вероятность события В, найденная в предположении, что событие А уже наступило. Теорема 2. Вероятность произведения двух зависимых событий A и B равна произведению вероятности одного из них на условную вероятность другого, найденного в предположении, что первое событие уже наступило. P(A⋅B)=P(A) ⋅PА (B) Два события называются совместными, если появление одного из них не исключает появление другого в одном и том же испытании. Два события называются противоположными, если в данном испытании они несовместны и одно из них обязательно происходит. Суммой событий A и B называется событие C=A+B, состоящее в наступлении, по крайней мере, одного из событий A или B, т.е. наступлении события А, или события В, или обоих этих событий вместе, если они совместны. Теорема 3. Вероятность суммы двух несовместных событий A и B равна сумме вероятностей этих событий: P(A+B)=P(A)+P(B) Следствие. Сумма вероятностей противоположных событий А и Ā равна единице: P(A)+P(Ā)=1 Вероятность суммы полной группы событий равна 1. Теорема 4. Вероятность суммы двух совместных событий A и B равна сумме вероятностей этих событий минус вероятность их произведения P(A+B) = P(A) + P(B) – P(A⋅B) Теорема 5 (Формула полной вероятности). Вероятность события A, которое может наступить лишь при условии появления одного из n попарно Copyright ОАО «ЦКБ «БИБКОМ» & ООО «Aгентство Kнига-Cервис»
  • 26. 26 несовместных событий B1,B2 ,…, Bn образующих полную группу, равна сумме произведений вероятностей каждого из этих событий на соответствующую условную вероятность события A: )()(...)()()()()( 21 21 APBPAPBPAPBPAP nBnBB ⋅++⋅+⋅= или ∑= = n i Bi APBPAP i 1 )()()( . Теорема 6 (Формула Бейеса). Если существуют n попарно несовместных событий B1,B2,…,Bn образующих полную группу, известны условные вероятности события А, то можно найти вероятности, того что событие А произошло при условии появления некоторого события Вk, можно найти по формуле: ( ) ( ) ( ) ( ) ( ) ( ) ( )APBPAPBP APBP BP n k BnB Bk kA ++ ⋅ = ...11 Примеры решения задач на вычисление вероятности по формулам полной вероятности и Бейеса см. в части 1 «Лекции» стр. 31-32. Пример 1: Вероятность попадания для первого стрелка 0,85, для второго – 0,8. Спортсмены независимо друг от друга сделали по одному выстрелу. 1) Найти вероятность того, что в мишень попадут оба стрелка? Решение. А – попал 1-ый стрелок B – попал 2-ой стрелок События А и В – независимые. Событие А⋅В состоит в том, что в результате испытания произошло и событие А и событие В, т.е. оба события. P(A)=0,85 P(B)=0,8. Формула: P(AB)=P(A) ⋅ P(B). P(AB)=0,85⋅0,8 Ответ: 0,68 2) Найти вероятность того, что промахнутся оба стрелка Решение. À – промахнулся 1-ый стрелок Â – промахнулся 2-ой стрелок Событие À является противоположным событию А, т.е. Р( À )=1-P(A)=1-0,85=0,15 Событие Â является противоположным событию В, т.е. Copyright ОАО «ЦКБ «БИБКОМ» & ООО «Aгентство Kнига-Cервис»
  • 27. 27 Р( Â )=1-P(В)=1-0,8=0,2. События À и Â независимые. Формула: P( ÀÂ )=P( À )⋅P( Â ). P( ÀÂ )=0,15⋅0,2 Ответ: 0,03 3) Найти вероятность того, что в мишень попадет хотя бы один стрелок Решение. А – попал 1-ый стрелок, B – попал 2-ой стрелок События А и В – совместные. Событие А+В состоит в том, что в результате испытания произошло или событие А, или событие В, или оба события. P(A)=0,85 P(B)=0,8. Формула: P(A+B) = P(A) + P(B) – P(A⋅B) P(A+B)=0,85+0,8-0,68 Ответ: 0,97 4) Найти вероятность того, что в мишень попадет ровно один стрелок. Решение. А – попал 1-й стрелок; À – промахнулся 1-й стрелок B – попал 2-й стрелок; Â – промахнулся 2-й стрелок Событие «в мишень попадет ровно один стрелок» означает, что попал 1-й стрелок и промахнулся 2-й или промахнулся 1-й и попал 2-й стрелок. P(A)=0,85 Р( À )=1-P(A)=1-0,85=0,15 P(B)=0,8 Р( Â )=1-P(В)=1-0,8=0,2. Формула: )()()()()( BPAPBPAPBABAP ⋅+⋅=⋅+⋅ =⋅+⋅ )( BABAP 0,85⋅0,2+0,15⋅0,8 Ответ: 0,29 Пример 2: В группе из 20 студентов: 5 – отличников, 10 – хорошо успевающих и 5 – занимающихся слабо. На предстоящем экзамене отличники могут получить только отличные оценки. Хорошо успевающие студенты могут получить с равной вероятностью хорошие и отличные оценки. Слабо занимающиеся студенты могут получить с равной вероятностью хорошие, удовлетворительные и неудовлетворительные оценки. Для сдачи экзамена приглашается наугад один студент. Найти вероятность того, что он получит хорошую или отличную оценку. Copyright ОАО «ЦКБ «БИБКОМ» & ООО «Aгентство Kнига-Cервис»
  • 28. 28 Решение. А – студент получил хорошую или отличную оценку B1 – приглашен студент – отличник Р(B1)=0,25 B2 – приглашен студент – хорошист Р(B2)=0,5 B3 – приглашен слабый студент Р(B3)=0,25 Формула: )()()()()()()( 321 321 APBPAPBPAPBPAP BBB ⋅+⋅+⋅= Найдем условные вероятности ),(1 APB ),(2 APB )(3 APB : 1)(1 =APB 1)(2 =APB 3 1 )(3 =APB В соответствии с формулой полной вероятности получаем: Р(А) = 0,25 ⋅1 + 0,5 ⋅ 1 + 0,25 ⋅1/3 Ответ: 0,8333 Пример 3: В пяти ящиках находятся одинаковые по весу и размерам шары. В двух ящиках – по 6 голубых и 4 красных шара (это ящик состава B1). В двух других ящиках (состав B2) – по 8 голубых и 2 красных шара. В одном ящике (состав B3) – 2 голубых и 8 красных шаров. Наудачу выбирается ящик и из него извлекается шар. Извлеченный шар оказался голубым. Какова вероятность того, что голубой шар извлечен из ящика первого состава? Решение. А – из ящика извлечен голубой шар B1 – шар извлечен из ящика первого состава 4,0 5 2 )( 1 ==BP B2 – шар извлечен из ящика второго состава 4,0 5 2 )( 2 ==BP B3 – шар извлечен из ящика третьего состава 2,0 5 1 )( 3 ==BP Формула: ( ) ( ) ( ) ( ) ( ) ( ) ( ) ( ) ( )APBPAPBPAPBP APBP BP BBB B A 321 1 321 1 1 ++ ⋅ = Найдем условные вероятности ),(1 APB ),(2 APB )(3 APB : 6,0 10 6 )(1 ==APB 8,0 10 8 )(2 ==APB 2,0 10 2 )(3 ==APB Copyright ОАО «ЦКБ «БИБКОМ» & ООО «Aгентство Kнига-Cервис»
  • 29. 29 В соответствии с формулой Бейеса найдем искомую вероятность: ( ) 2,02,08,04,06,04,0 6,04,0 1 ⋅+⋅+⋅ ⋅ =BPA Ответ: 0,4 Задания для самостоятельного выполнения Задание 1. В урне 40 шаров: 15 голубых, 5 зеленых и 20 белых. Какова вероятность того, что из урны будет извлечен цветной шар? A – __________________________________________ B – __________________________________________ А⋅ В зависимые независимые А+В совместные несовместные Формула: ________________________________ Ответ: _____________ Задание 2. В шкатулке 6 синих и 4 красных катушки ниток. Из нее извлекают подряд две катушки. Какова вероятность, что обе вынутые катушки с синими нитками? A – __________________________________________ B – __________________________________________ А⋅ В зависимые независимые А+В совместные несовместные Формула: _________________________________ Ответ:______________ Задание 3. Найти вероятность того, что наудачу взятое двухзначное число окажется кратным 2 или 7. Задание 4. В урне 15 шаров, из которых 5 голубых и 10 красных. Из нее последовательно вытаскивают два шара. Какова вероятность, что первым извлеченным окажется красный шар, а вторым – голубой? Задание 5. Студент знает 20 из 25 вопросов программы. Определить вероятность того, что 1) студент знает все три предложенных ему вопроса; 2) студент не знает ни один из трех предложенных ему вопроса. Задание 6. Для приема экзамена заготовлено 50 задач. 20 задач из раздела «А», 20 – «В», 10 – «С». Для сдачи экзамена студент должен решить первую попавшуюся задачу. Какова вероятность сдать экзамен, если он умеет решать все задачи из раздела «А», 10 из «В» и 5 из «С»? Какова вероятность, что ему попалась задача из раздела «А»? Copyright ОАО «ЦКБ «БИБКОМ» & ООО «Aгентство Kнига-Cервис»
  • 30. 30 Вопросы для самоконтроля 1. Чему равна вероятность суммы двух совместных событий? 2. Чему равна вероятность суммы двух несовместных событий? 3. Чему равна сумма вероятностей противоположных событий? 4. Как определить вероятность произведения двух независимых событий? 5. Как связаны между собой формула Бейеса и формула полной вероятности? 6. В каких случаях применяют формулу Бейеса? 7. В каких случаях применяют формулу полной вероятности? Индивидуальные задания 1. Бросается один раз игральная кость. Определите вероятность того, что выпадет 1 или 6 очков. 2. В школу поступили партии книг, выпущенные двумя издательствами: 25% – из первого издательства и 75% со второго. В продукции первого издательства 60% учебников по математике, второго – 80%. Какова вероятность, что наудачу взятая книга окажется учебником по математике? 3. В группе спортсменов 20 лыжников и 8 бегунов и 7 пловцов. Вероятность выполнить квалификационную норму для лыжника равна 0,8, для бегуна – 0,7 для пловца – 0,85. Спортсмен выполнил норму. Какова вероятность того, что он лыжник? 4. В лабиринте на правильном маршруте имеются три развилки, на каждой из которых нужно выбирать одно из двух направлений. Какова вероятность, что испытуемый пройдет лабиринт с первой попытки? 5. Из 50 табуреток 18 изготовлены учениками 7 «А» класса, 20 – учениками 7 «Б», остальные – в 7 «В». Ученики 7 «Б» и 7 «В» обычно выполняют качественную продукцию с вероятностью 0,9. В продукции учеников 7 «А» бракованные изделия составляют 15%. Взятая наудачу табуретка оказалась качественной. Какова вероятность того, что табуретка изготовлена учениками 7 «Б» класса? 6. Какова вероятность, что при троекратном бросании монеты выпали только орлы? 7. Учебное заведение получает канцелярские товары от трех магазинов: 1– 30 %; 2 – 55 %; 3 – 15 %. Известно, что брак составляет: 1 – 5%; 2 – 6%; 3 – 10%. Полученные канцелярские товары хранятся на общем складе. Какова вероятность того, что наугад взятый товар оказался бракованным? Copyright ОАО «ЦКБ «БИБКОМ» & ООО «Aгентство Kнига-Cервис»
  • 31. 31 Случайные величины Основные умения: находить характеристики случайных величин; применять законы распределения случайных величин к решению профессиональных задач. Рекомендуемая литература: [7, 12, 15]. Понятие случайной величины Случайной величиной (СВ) называется переменная величина, которая в зависимости от исхода испытания случайно принимает одно значение из множества возможных, зависящих от случая. Случайная величина, принимающая различные значения, которые можно записать в виде конечной или бесконечной последовательности, называется дискретной случайной величиной. Случайная величина, которая может принимать все значения из некоторого промежутка, называется непрерывной случайной величиной. Представление о случайных событиях, результатом которых являются числовые значения см. в части 1 «Лекции» стр. 34. Закон распределения случайных величин Законом распределения дискретной СВ называется соответствие между значения x1, x2, …, xn этой величины и их вероятностями р1, р2, … рn. Закон распределения дискретной СВ может быть задан таблично или аналитически (то есть с помощью формул). Х х1 х2 … хn-1 хn Р р1 р2 … рn-1 рn где x1, x2, …, xn – случайные величины, соответствующие полной группе событий: р1+р2+…+ рn = 1. Математическим ожиданием М(Х) дискретной случайной величины Х называется сумма произведений всех возможных значений величины Х на соответствующие вероятности: ∑ +++== = n i nnii pxpxpxpxXM 1 2211 ...)( . Дисперсией D(X) дискретной случайной величины Х называется математическое ожидание квадрата отклонения случайной величины Х от ее математического ожидания: D(X) = M [(X – M(X))2 ]. Вычисления удобней производить по формуле: D(X) = M(X2 ) – M2 (X). Copyright ОАО «ЦКБ «БИБКОМ» & ООО «Aгентство Kнига-Cервис»
  • 32. 32 Средним квадратическим отклонением σ(Х) случайной величины Х называется корень квадратный из ее дисперсии: )()( XDX =σ . Возможности применения законов распределения случайных величин и их характеристик к исследованию процессов реального мира представлены в части 1 «Лекции» стр. 36-40. Биномиальное распределение Пусть производится n испытаний, причем вероятность появления события А в каждом испытании равна р, то вероятность его ненаступления равна q = 1 – р. Найдем вероятность того, что при n испытаниях событие А наступит т раз (m ≤ n). Формула Бернулли: ( ) ( ) mnm n qp n-mm n mP − = !! ! Закон биномиального распределения Х 0 1 … m … n р qn npqn-1 Cn m pm qn-m pn Пример 1: Пусть всхожесть семян определенного растения составляет 90 %. Найти вероятность того, что из четырех посеянных семян взойдут: а) три; б) не менее трех. Напишите в виде таблицы закон распределения случайной величины Х – всхожесть семян. Решение. а) в данном случае n = 4; m = 3; p = 0,9; q = 1 – p; q = 0,1. Применяя формулу Бернулли, получим: ( ) ( ) ( ) ;2916,01,09,0 1!!3 !4 !! ! 3 ==⋅= −mnm n qp n-mm n mP б) искомое событие А состоит в том, что из четырех семян взойдут или три, или четыре. По теореме сложения вероятностей получим: ;(4)(3))( 44 PPAP += ( ) ;6561,09,0(4) 4 4 ==P 6561,02916,0)( +=AP =0,9477 Закон распределения: n = 4; p = 0,9; q = 0,1: для m = 0 (семена не прорастут) P4(0)=qn =(0,1)4 =0,0001; для m = 1 (прорастет 1 семя) P4(1)=npqn-1 =4⋅0,9⋅(0,1)3 =0,0036; для m = 2 P4(2)=С4 2 pqn-1 =6⋅(0,9)2 ⋅(0,1)2 =6⋅0,81⋅0,01=0,0486; для m = 3 P4(3)= 0,2916; для m = 4 (прорастут все семена) P4(4)=0,6561. Copyright ОАО «ЦКБ «БИБКОМ» & ООО «Aгентство Kнига-Cервис»
  • 33. 33 Х 0 1 2 3 4 P 0,0001 0,0036 0,0486 0,2916 0,6561 Проверка: 0,0001+0,0036+0,0486+0,2916+0,6561=1. В MS Excel функция БИНОМРАСП применяется для вычисления вероятности в задачах с фиксированным числом тестов или испытаний, когда результатом любого испытания может быть только успех или неудача. Функция использует следующие параметры: БИНОМРАСП (число_успехов; число_испытаний; вероятностъ_успеха; интегральная), где число_успехов – количество успешных испытаний; число_испытаний – число независимых испытаний (число успехов и число испытаний должны быть целыми числами); вероятность_ успеха – вероятность успеха каждого испытания; интегральная – логическое значение, определяющее вид функции: если параметр имеет значение ИСТИНА (=1), то считается интегральная функция распределения (вероятность того, что число успешных испытаний не менее значения число_ успехов); если параметр имеет значение ЛОЖЬ (=0), то вычисляется значение функции плотности распределения (вероятность того, что число успешных испытаний в точности равно значению аргумента число_ успехов). Пример 2: Какова вероятность того, что а) трое из четырех новорожденных будут мальчиками? б) не более трех из четырех новорожденных будут мальчиками? Решение. 1. Устанавливаем табличный курсор в свободную ячейку, например в А1. Здесь должно оказаться значение искомой вероятности. 2. Для получения значения вероятности воспользуемся функцией: нажимаем на панели инструментов кнопку Вставка функции fx. 3. В появившемся диалоговом окне Мастер функций – шаг 1 из 2 слева в поле Категория указаны виды функций. Выбираем Статистическая. Справа в поле Функция выбираем функцию БИНОМРАСП и нажимаем на кнопку ОК. Появляется диалоговое окно функции. В поле Число_s вводим с клавиатуры количество успешных испытаний (3). В поле Испытания вво- дим с клавиатуры общее количество испытаний (4). В рабочее поле Вероятность_s вводим с клавиатуры вероятность успеха в отдельном испытании (0,5). В поле Интегральная вводим с клавиатуры вид функции распределения – интегральная (0). Нажимаем на кнопку ОК. а) в ячейке А1 появляется искомое значение вероятности р = 0,25. Ровно 3 мальчика из 4 новорожденных могут появиться с вероятностью 0,25. Copyright ОАО «ЦКБ «БИБКОМ» & ООО «Aгентство Kнига-Cервис»
  • 34. 34 б) Если выяснить вероятность того, что появится не менее трех мальчиков, то в этом случае в рабочее поле Интегральная вводим 1 (вид функции распределения интегральная). Вероятность этого события будет равна 0,9375. Нормальное распределение Нормальное распределение – это совокупность объектов, в которой крайние значения некоторого признака – наименьшее и наибольшее – появ- ляются редко; чем ближе значение признака к математическому ожиданию, тем чаще оно встречается. Закон распределения вероятностей непрерывной случайной величины Х называется нормальным, если ее дифференциальная функция f(x) определяется формулой: ( ) ( ) , 2 1 2 2 2σ πσ ax exf − − = где а совпадает с математическим ожиданием величины Х: а=М(Х), параметр σ совпадает со средним квадратическим отклонением величины Х: σ = σ(Х). Диаграмма нормального распределения симметрична относительно точки а (математического ожидания). Медиана, среднее арифметическое нормального распределения равны тоже а. При этом в точке а функция f(x) достигает своего максимума, который равен πσ 2 1 . Пример 3: График плотности вероятности нормального распределения непрерывной величины X изображен на рисунке. Определите математическое ожидание, среднее квадратическое отклонение и максимальное значение дифференциальной функции распределения. Решение. По графику можно найти максимальное значение дифференциальной функции распределения, оно составляет 0,2. Функция достигает максимума в точке x=5, следовательно, математическое ожидание M(X)=5. В точке максимума функция плотности вероятности примет вид: ( ) πσ 2 1 =xf , следовательно, 2 5,22,0 1 14,322,0 1 2)( 1 ≈ ⋅ = ⋅ == π σ xf Copyright ОАО «ЦКБ «БИБКОМ» & ООО «Aгентство Kнига-Cервис»
  • 35. 35 В MS Excel для вычисления значений нормального распределения используются функция НОРМРАСП, которая вычисляет значения вероятности нормальной функции распределения для указанного среднего и стандартного отклонения. Функция имеет параметры: НОРМРАСП (х; среднее; стандартное_откл; интегральная), х – значения выборки, для которых строится распределение; среднее – среднее арифметическое выборки; стандартное_откл – стандартное отклонение распределения; интегральная – логическое значение, определяющее форму функции. Если интегральная имеет значение ИСТИНА(1), то функция НОРМРАСП возвращает интегральную функцию распределения; если это аргумент имеет значение ЛОЖЬ (0), то вычисляется значение функции плотности распределения. Если среднее = 0 и стандартное_откл = 1, то функция НОРМРАСП возвращает стандартное нормальное распределение. Пример 4: Составить дифференциальную функцию распределения непрерывной величины X, если известно, что величина распределена по нормальному закону с параметрами: математическое ожидание равно -2, а среднее квадратическое отклонение 2. Изобразить полученную функцию с помощью MS Excel. Решение. Дифференциальная функция распределения непрерывной величины X, распределенной по нормальному закону, имеет вид: ( ) ( ) , 2 1 2 2 2σ πσ ax exf − − = где а – математическое ожидание; σ – среднее квадратическое отклонение. По условию задачи математическое ожидание а=-2; среднее квадратическое отклонение σ=2, следовательно ( ) ( ) 8 2 2 22 1 + − = x exf π . Для построения графика необходимо выбрать начальное значение для переменной x. Серединное значение совпадает с математическим ожиданием а, начальное значение отстоит от серединного не менее чем на σ, поэтому примем начальное значение x=-5. Запишем в ячейку A1 значение -5, в ячейку А2 – формулу =А1+0,2 и «протянем» эту формулу до ячейки А31, в которой получится значение 1. В ячейку B1 внесем формулу: =1/(2*КОРЕНЬ(2*ПИ()))*EXP(-((A1+2)^2)/8) и «протянем» эту формулу до 0 0,05 0,1 0,15 0,2 0,25 -8 -6 -4 -2 0 2 4 Copyright ОАО «ЦКБ «БИБКОМ» & ООО «Aгентство Kнига-Cервис»